You are on page 1of 149

CONTENTS

CHAPTER (1)
PROGRAM OF MATRICES
1-1 Introduction ………………………………... 1
1-2 PROGRAMMING LANGUAGES………... 2
1-3 ELEMENTARY MATRIX OPERATION . 3
1-3-1 Definitions and Rules………………………………. 3
1-3-2 Transpose of Rectangular Matrix ……………….. 7
1-3-3 Transpose of Square Matrix ……………………… 8
1-3-4 Addition and Subtraction of Matrices……………. 8
1-3-5 Program of Addition and Subtraction of Matrices 9
1-3-6 Multiplication of Matrices 10
1-3-7 Program of Multiplication of Matrices 11
1-3-8 Program of Inverse of Matrices 12
Solved Problems 13
Sheet ( 1 ) 16

CHAPTER (2)
ANALYSIS OF INDETERMINATE
STRUCTURES
2-1 INTRODUCTION 17
2-2 DEFINITION 17
2-3 DEGREE OF INDETERMANCY 20
2-4 KINEMATIC INDETERMANCY 23
2-5 ANALYSIS OF STATICALLY INDETERMINATE 24
STR.
2-6 FORCE – DISPLACEMENT RELATIONS 25
2-7 DESCRIPTION OF METHODS 25
2-7-1 Solution by flexibility method 26
2-7-2 Solution by stiffness method 27
2-7-3 Comparison between the flexibility and 28
stiffness method
Sheet ( 2 )

CHAPTER (3)
SLOPE DEFLECTION METHOD
3-1 Introduction …………………………………. 30
3-2 Assumption in slope deflection ……………... 31
3-2-1 Degree of Freedom ………………………….. 31
3-3 Sign Conventions …………………………… 34
3-4 Fixed End Moments ………………………… 35
3-5 Derivation of Slope Deflection Equation …… 36
3-5-1 Examples ……………………………………. 38
3-6 Frames with a single degree of freedom in translation
(Examples) ………………………. 51
3-7 Frames with Multiple degrees of freedom in
58
translation (Examples) ………………………..
Sheet ( 3 ) 74
CHAPTER (4)
MATRIX APPROCH 1
(SLOPE DEFLECTION IN MATRIX FORM)
4-1 Introduction ………………………………….. 76
4-2 Members Without Relative Displacements……... 76
4-3 Solved Examples…………………………… 79
4-4 Effect of Members With Moments Releases...…. 85
Sheet ( 4 ) …………………………………….. 86
CHAPTER (5)
STIFNESS MATRIX METHOD
5-1 Definition……………….…………………... 87
5-2 Step 1: Modeling………………………………. 89
5-3 Step 2: Load Vector………………………….. 94
5-4 Step 3: Stiffness Matrix……………………… 95
5-5 Step 4: Equilibrium Equation ……… 97
5-6 Step 5: Solve The Equilibrium Equation … 97
5-7 Step 6 : Internal Forces……… ………. 98
CHAPTER (6)
ANALYSI OF PLANE TRUSSES BY USING
STIFNESS MATRIX METHOD
6-1 Modeling………………………………. 99
6-2 Load Vector………………………….. 102
6-3 Stiffness Matrix……………………… 104
6-4 Equilibrium Equation And Internal Forces 106
6-5 Solved Examples 107
Sheet ( 5 )……… ………. 118
CHAPTER (7)
ANALYSI OF PLANE FRAMES BY USING
STIFNESS MATRIX METHOD
7-1 Modeling………………………………. 120
7-2 Load Vector………………………….. 123
7-3 Stiffness Matrix……………………… 133
7-4 Equilibrium Equation And Internal Forces 137
7-5 Solved Examples 138
Sheet ( 6 )……… ………. 158
References …………………………………… 160
Examinations 161
CHAPTER 1

PROGRAM OF MATRICES

1-1- INTRODUCTION

A definition of engineering is the science by which the properties


of matter and sources of energy in nature are made useful to man.
Thus an engineer will have to study the properties and behavior of
physical systems, applying his knowledge to attain purposes useful to
society.

Such activity can be considered as consisting of the study and


solution of real physical. The complexities involved in dealing with
all the parameters relative to the properties of a real problem will, on
many occasions, force him to study an equivalent engineering
problem which can be mathematically defined. He will try to solve
such a problem according to his knowledge and experience, and using
the computational tools at hand.

When we think of today’s engineering problems, such as the


construction of nuclear reactors, space satellites, oil rigs in the sea,
etc. it is clear that they must be analyzed using very relined discrete
models, so as to closely approximate their real behavior.

Fortunately, with the help of the modern computer, it is possible


today to treat discrete models with up to several thousands of degrees
of freedom. Thus, the need for formulating larger and more complex
discrete models, and the availability of computers which make
feasible their analysis, are the causes of the considerable interest
produced in relation to numerical methods and computer analysis
techniques.
1-2- PROGRAMMING LANGUAGES

A computer can perform elementary operations using stored


information. The user can order the computer to perform an operation,
giving the proper instruction, as a part of his program. The set of all
the instructions which a computers accepts constitutes the machine
language, or absolute language of that computer. The machine
instructions are always expressed by numeric codes. Each instruction
normally consists of one operation code, and one or more parameters
or arguments. For instance, the sequence can represent two
instructions written according to a given machine language.

He first step in programming the solution of a problem, is to


make an analysis of the problem. The problem is studied in all its
detail, and an appropriate solution scheme is selected. Then the
computer solution to be implemented is schematized graphically, by
means of a diagram called a flow chart. These diagrams can be
detailed to a greater or leaser extent, according to circumstances, but
they are fundamental to the analysis and understanding of all aspects
of the computer solution. The person in charge of this task is normally
called an analyst.

Once the flow-chart is completed, the computer solution is coded,


writing the computer program using the language selected. This task
is carried out by a programmer.

With the program registered in some information support, and the


data and proper control commands added, the program is tested by
submitting it for processing. At the beginning the program will
include some errors. These can be syntax errors and logic errors.
Syntax errors appear because some of the rules of the language are
violated. Normally These can be easily detected and corrected. Logic
errors correspond to the case that a syntax errors-free program is
submitted for execution, and it does not produce the desired results.
The following situations can be encountered.

1. Execution of the program stops before producing results.


2. Execution of the program does not stop and results are not
produced.
The program is finally discontinued after exceeding the
allowed time.
3. Results are produced, but they are incorrect.
4. The program works successfully.

1-3- ELEMENTARY MATRIX OPERATION


1-3-1 Definitions and Rules
Matrix algebra is the set of rules which defines operations
performed with matrices. A matrix is a bi-dimensional array of
numbers organized according to “ n “ rows and “ m “ columns, which
we refer to as a matrix of order “ n x m “. In what follows, a matrix
will be indicated by bold type. Thus, R will be a matrix, while T is a
scalar number. The elements or coefficients of a matrix will be two
sub-indices, indicating row and column position, respectively. For
instance

[A] = ⎡⎢
a11 a12 a13 ⎤
(1-1)
⎣ a 21 a 22 a 23 ⎥⎦

is a rectangular matrix , of order 2 x 3, that is having 2 rows and 3


columns. The sub-indices of the coefficients indicate their position
within the matrix. Thus, the coefficient a12 is placed in the first row
and the second column.

A matrix having the same number of rows and columns is called


a square matrix, such as
⎡ b11 b12 b13 ⎤
[B] = ⎢⎢b21 b22 b23 ⎥⎥ (1-2)
⎢⎣b31 b32 b33 ⎥⎦

which is a square matrix of 3 rows and 3 columns, or of order 3 x 3.


In case such as this, when speaking specifically of square matrices,
we can simply say that B is of order 3. The coefficients b11, b22, and
b33 or in general bij for i = j, are called the coefficients of the principal
or main diagonal of the matrix.

Several types of square matrices can be considered. When the


coefficients of a square matrix B satisfy the condition

bij = bji , for i = 1,2,….,n and j = 1,2,….,n (1-3)

this called a symmetric matrix. In particular, if B is a symmetric


matrix of order 3, it will have the following structure

⎡ b11 b12 b13 ⎤


[B] = ⎢⎢b12 b22 b23 ⎥⎥ (1-4)
⎢⎣ b13 b23 b33 ⎥⎦

On the other hand, skew-symmetric matrix is such that


bij = -bji , for i ≠ j & bij = 0, for i = j (1-5)
for example,

⎡ 0 b12 b13 ⎤
[B] = ⎢⎢ − b12 0 b23 ⎥⎥ (1-6)
⎢⎣ − b13 − b23 0 ⎥⎦

when all the coefficients of a square matrix B are null, except for
the main diagonal, such as,

⎡ b11 0 0⎤
[B] = ⎢⎢ 0 b22 0 ⎥⎥ (1-7)
⎢⎣ 0 0 b33 ⎥⎦
this is called a diagonal matrix. If, in particular, all the coefficients
of the main diagonal are equal to 1, the matrix is called a unit matrix,
and is symbolized I. For instance, the following is the unit matrix of
order 3.

⎡1 0 0 ⎤
[I ] = ⎢⎢0 1 0⎥⎥ (1-8)
⎢⎣0 0 1⎥⎦

A matrix having only one column is called a column vector.


Normally, the coefficients of a column vector will be
given only one subscript, corresponding to the row position, and will
be enclosed by curled brackets instead of square brackets, placed
vertically, as in

⎧ c1 ⎫
{C} = ⎪⎨c 2 ⎪⎬ (1-9)
⎪c ⎪
⎩ 3⎭

where C is a column vector of order 3.

When a matrix has only one row it will be called a row vector and
again, each of its coefficients will be given only one subscript. For
instance,

{D} = {d1 d2 d3 d4} (1-10)

is a row vector of order 3.

In what follows the use of the term “vector” alone will always be
taken to mean “column vector”.

The transpose of a matrix is obtained by interchanging rows and


columns, of the matrix, and is indicated by an upper-index T. For
instance, the transpose of A, given by eq. (1-1) is
⎡ a11 a 21 ⎤
[A] = ⎢⎢ a12 a 22 ⎥⎥ (1-11)
T

⎢⎣ a13 a 23 ⎥⎦

while the transpose of matrix B, given by eq. (1-2) is

⎡b11 b21 b31 ⎤


[B] = ⎢⎢b12 b32 ⎥⎥ (1-12)
T
b22
⎢⎣b13 b23 b33 ⎥⎦

It can be noticed that the transpose of a column vector is a row


vector, and vice versa. Thus, the transpose of column vector, given by
eq. (1-9) is

{C}T = {c1 c2 c3 } (1-13)

While the transpose of row vector, given by eq. (1-10) is

⎧ d1 ⎫
⎪d ⎪
⎪ ⎪
{D}T = ⎨ 2⎬ (1-14)
⎪d 3 ⎪
⎪⎩d 4 ⎪⎭

The transpose of the transpose gives the original matrix, that is

([A] )T T
= [A] (1-15)

For symmetric matrix, the transpose of the matrix is the original matrix,

[B] = [B]T (1-16)


Another special matrix which can be defined as the null matrix,
represented by 0, whose coefficients are all equal to zero. For
instance,

[0] = ⎡⎢
0 0 0⎤
⎥ (1-17)
⎣0 0 0 ⎦

is a null matrix of order 2 x 3

1-3-2 Transpose of Rectangular Matrix


The following routine is a very simple example of a computer
program operating with matrices which can be used to obtain the
transpose of a rectangular matrix.

C This program computes the transpose of a matrix “A” of order n


xm
DIMENSION A(10,10),B(10,10)
DO 10 I=1,n
DO 10 J=1,m
10 B(J,I) = A(I,J)
END

where the array A contains the matrix to be transposed, the integer n


contains the number of rows, and the integer m contains the number
of columns.
1-3-3 Transpose of Square Matrix
In the case of a square matrix it is possible to transpose the matrix
in itself, without creating a new one. The following routine receives
the original matrix in the array A, of order n, and after operating
returns the transposed matrix in the same array A. Note that an
auxiliary variable S is used to allow for the transposition in place.

C This program computes the transpose of a square matrix “A” of order n

DIMENSION A(10,10)
n1 = n - 1
DO 10 I=1,n1
I1 = I + 1
DO 10 J=I1,n
S = A(1,J)
A(I,J) = A(J,I)
10 A(J,I) = S
END

1-3-4 Addition and Subtraction of Matrices

Two different matrices can be added or subtracted, provided they


are of the same order. Let us consider the following two matrices

[A] = ⎡⎢
a13 ⎤
[B] = ⎡⎢
a11 a12 b11 b12 b13 ⎤
, (1-18)
⎣ a 21 a 22 a 23 ⎥⎦ ⎣b21 b22 b23 ⎥⎦

By definition the sum of A and B will give another matrix C, also of


the same order, i.e.

[C ] = ⎡⎢
c11 c12 c13 ⎤
(1-19)
⎣ c 21 c 22 c 23 ⎥⎦
where coefficients are
c11 = a11 + b11 c12 = a12 + b12 c13 = a13 +
b13
c21 = a21 + b21 c22 = a22 + b22 c23 = a23 +
b23
In general,
cij = aij + bij for i = 1,2,….,n and j = 1,2,….,m (1-20)

1-3-5 Program of Addition and Subtraction of Matrices

The following routine can be used for addition and subtraction of


matrices. The arrays A and B contain the matrices to be added or
subtracted, which of order n x m. The array C will contain the result
matrix. When the integer L = 1 is specified the routine will perform
the addition operation, but when L = 2 the routine will perform the
subtraction operation.

C This program computes the matrix operation


C C=A+B when L = 1 and
C C=A-B when L = 2
C n : number of rows
C m : number of columns
DIMENSION A(10,10), B(10,10), C(10,10)
DO 10 I=1,n
DO 10 J=1,m
GO TO (2,4), L
2 C(I,J) = A(I,J) + B(I,J)
GO TO 10
4 C(I,J) = A(I,J) - B(I,J)
10 CONTINUE
END
1-3-6 Multiplication of Matrices

Multiplication of matrices can also be performed. Let us consider


a matrix A of order n x m, and matrix B of order m x n, such as

⎡ b11 b12 ⎤
[A] = ⎡⎢ 11
a13 ⎤
[B ] = ⎢⎢b21 b22 ⎥⎥
a a12
, (1-21)
⎣ a 21 a 22 a 23 ⎥⎦
⎣⎢b31 b32 ⎦⎥

The matrix multiplication equation

[C ] = [A][B] (1-22)

gives as a result a new matrix with coefficients:

c11 = a11 b11 + a12 b21 + a13 b31 c12 = a11 b12 + a12
b22 + a13 b32
c21 = a21 b11 + a22 b21 + a23 b31 c22 = a21 b12 + a22
b22 + a23 b32

or, in general:

m
cij = ∑ aik bkj (1-23)
k =1

It is easily seen that two matrices can be multiplied only if the


number of columns of the first matrix is equal to the number of rows
of the second matrix. The result matrix will have the same number of
rows of the first matrix and the same number of columns of the
second matrix. Notice that according to its definition, matrix
multiplication is not commutative, i.e.,
[A] [B] ≠ [B] [A] (1-24)

and that, in particular, if the left–hand side is defined, the right-hand


side may be undefined unless the number of rows of matrix A is
equal to the number of columns of matrix B. Even in that cases, the
results of the left-hand side, and the right-hand side will be different,
excluding very particular cases.

1-3-7 Program of Multiplication of Matrices

The multiplication of two matrices can be readily programmed.


The following routine receives the first matrix of order n x m, in the
array A, and the second matrix of order m x h, in the array B. The
result matrix , of order n x h, will be stored in the array C. Note that
since the implementation of eq. (1-23) requires accumulating the
partial multiplications in the array C, we have been careful in
previously setting cij = 0.

C This program computes the matrix operation


C C=A*B
C n : number of rows of A and C
C m : number of columns of A and rows of B
C h : number of columns of B and C
DIMENSION A(10,10), B(10,10), C(10,10)
DO 10 I=1,n
DO 10 J=1,h
C(I,J) = 0
DO 10 K=1,m
10 C(I,J) = C(I,J) + A(I,K) * B(K,J)
END
1-3-8 Program of Inverse of Matrices
The inverse of matrix C is the matrix which satisfy the following relation

[C] [C]-1 = [I]


The following routine can be used for computing the matrix inverse [C]-1.

C This program computes the matrix operation


DIMENSION C(5,5), ST(5,10), C1(5,5)
DO 2 I=1,5
DO 2 J=1,5
2 ST(I,J) = C(I,J)
M1 = 6
K=
DO 3 I=1,5
DO 3 J=6,10
IF (I.EQ.(J-5)) GO TO 4
ST(I,J) = 0.0
GO TO 3
4 ST(I,J) = 1
3 CONTINUE
L=1
10 DO 5 J=1,K
IF (L.EQ.J) GO TO 6
ST(L,J) = ST(L,J) / ST(L,L)
5 CONTINUE
ST(L,L) = 1
DO 6 J=1,5
IF(J.EQ.L) GO TO 6
IF(ST(I,J).EQ.0.0) GO TO 6
DO 7 M=1,K
IF(L.EQ.M) GO TO 7
ST(J,M) = ST(J,M) – ST(J,L) * ST(L,M)
7 CONTINUE
ST(J,L) = 0.0
6 CONTINUE
IF(L.EQ.6) GO TO 8
GO TO 10
8 DO 9 I=1,5
DO 9 J=1,5
9 C1(I,J) = ST(I,J+5)
END
Solved Problems
1- For the following matrices, evaluate the following
expressions if possible. If not possible, state the reason:

a) 3[D] – 4[E]
]
b) [A] x [C]

c) [A] x 0.5[B]

⎡ 7 − 5⎤ ⎡ 7 −6 3 6 − 1⎤
−2
[B] = ⎡⎢
4 7 ⎤
[A] = ⎢⎢ 1 5 ⎥⎥ [C ] = ⎢⎢ 2 8 − 6 4 2 ⎥⎥
⎣ 6 3 − 0.5⎥⎦
⎢⎣ − 1 4 ⎥⎦ ⎢⎣ − 1 3 0 5 3 ⎥⎦

⎡ 7 −5 3 ⎤ ⎡ 6 −5 2⎤
⎢1 5 − 1⎥⎥ ⎢ 8 −4 − 1⎥⎥
[D] = ⎢ [E ] = ⎢
⎢−1 4 0⎥ ⎢− 3 0 2⎥
⎢ ⎥ ⎢ ⎥
⎣0 3 1⎦ ⎣0 3 1 ⎦

Solution
a) 3 [D] – 4 [E]

⎡ 7 −5 3 ⎤ ⎡ 6 −5 2⎤
⎢1 ⎥
5 − 1⎥ ⎢ 8 −4 − 1⎥⎥
3 [ D] − 4 [ E ] = 3 ⎢ −4⎢
⎢−1 4 0⎥ ⎢− 3 0 2⎥
⎢ ⎥ ⎢ ⎥
⎣0 3 1⎦ ⎣0 3 1 ⎦

⎡ 21 − 15 9 ⎤ ⎡ 24 − 20 8 ⎤
⎢ 3 15 − 3 ⎥⎥ ⎢ 32 − 16 − 4 ⎥⎥
=⎢ − ⎢
⎢ −3 12 0 ⎥ ⎢ − 12 0 8 ⎥
⎢ ⎥ ⎢ ⎥
⎣ 0 9 3 ⎦ ⎣ 0 12 4 ⎦

⎡ −3 5 1 ⎤
⎢− 29 31 1 ⎥⎥
=⎢
⎢ 9 12 −8 ⎥
⎢ ⎥
⎣ 0 −3 −1 ⎦
b) [A]3x2 x [C]3x5

‫ﻏﻴﺮ ﻣﻤﻜﻨﺔ ﻟﻌﺪم ﺗﺴﺎوي ﻋﺪد أﻋﻤﺪة اﻟﻤﺼﻔﻮﻓﺔ اﻷوﻟﻰ ﻣﻊ ﻋﺪد ﺻﻔﻮف اﻟﻤﺼﻔﻮﻓﺔ اﻟﺜﺎﻧﻴﺔ‬

c) 5[A] x [B]

⎡ 7 − 5⎤
⎡ 4 −2 7 ⎤
5 [ A] x 0.5 [ B ] = 5 ⎢⎢ 1 5 ⎥⎥ x 0.5 ⎢
⎣ 6 3 − 0.5⎥⎦
⎢⎣ − 1 4 ⎥⎦
⎡ 35 − 25⎤
⎡ 2 −1 3.5 ⎤
= ⎢⎢ 5 25 ⎥⎥ x ⎢
⎣ 3 1.5 − 0.25⎥⎦
⎢⎣ − 5 20 ⎥⎦
⎡ −5 − 150 128.75⎤

= ⎢ 85 − 72.5 11.25 ⎥⎥
⎢⎣ 50 35 − 22.5 ⎥⎦

2) Find the inverse of the following matrix:

⎡ 7 −5 3 0 −1 2⎤
⎢1 5 −1 7 2 2 ⎥⎥

⎢−1 4 0 5 1 3⎥
[A]6 x 6 =⎢ ⎥
⎢0 3 1 2 0 4⎥
⎢ 0 −1 7 3 0 − 1⎥
⎢ ⎥
⎣⎢ 2 − 2 3 −2 1 0 ⎦⎥

Solution

[A]−1 = 1
A
[ ]
Aadj

By using Saraus method, we can get A


7 −5 3 0 −1 2 7 −5 3 0 −1
1 5 −1 7 2 2 1 5 −1 7 2
−1 4 0 5 1 3 −1 4 0 5 1
0 3 1 2 0 4 0 3 1 2 0
0 −1 7 3 0 −1 0 −1 7 3 0
2 −2 3 −2 1 0 2 −2 3 −2 1
− − − − − − + + + + + +
∴ A = 0 + 0 + 0 + 0 – 84 + 24 – (-40) – (-392) – 0 – 0 – 0 – 0 = 372

Q A ≠0 ∴ [A] is nonsingular matrix.

⎡ − 10.45 54.68 − 16.74 − 314.34 − 16.36 − 29.64 ⎤


⎢ − 116.58
⎢ 141.36 − 15.54 − 21.94 41.29 98.58 ⎥⎥
⎢− 209.06 305.78 − 66.58 − 116.51 − 240.08 − 187.78 ⎥
∴ Matrix of min ors = ⎢ ⎥
⎢ − 94.41 193.47 − 62.27 − 88.53 165.91 − 23.06 ⎥
⎢ 4.46 − 29.64 45.79 − 13.46 − 59.89 40.17 ⎥
⎢ ⎥
⎢⎣ 13.39 − 31.62 − 18.56 − 40.54 − 198.46 39.5 ⎥⎦

⎡ − 10.45 − 54.68 − 16.74 314.34 − 16.36 29.64 ⎤


⎢ 116.58
⎢ 141.36 15.54 − 21.94 − 41.29 − 98.58 ⎥⎥
⎢− 209.06 − 305.78 − 66.58 116.51 240.08 − 187.78 ⎥
∴ Co − factor matrix = ⎢ ⎥
⎢ 94.41 193.47 62.27 − 88.53 − 165.91 − 23.06 ⎥
⎢ 4.46 29.64 45.79 13.46 − 59.89 − 40.17 ⎥
⎢ ⎥
⎣⎢ − 13.39 − 31.62 18.56 − 40.54 198.46 39.5 ⎦⎥

⎡ − 10.45 116.58 − 209.06 94.41 4.46 − 13.39 ⎤


⎢ − 54.68
⎢ 141.36 − 305.78 193.47 29.64 − 31.62 ⎥⎥
⎢ − 16.74 − 66.58 18.56 ⎥
[ ]
∴ Aadj = [ A ]T =⎢
15.54
− 21.94
62.27
− 88.53
45.79
− 40.54 ⎥

⎢ 314.34 116.51 13.46
⎢ − 16.36 − 41.29 240.08 − 165.91 − 59.89 198.46 ⎥
⎢ ⎥
⎢⎣ 29.64 − 98.58 − 187.78 − 23.06 − 40.17 39.5 ⎥⎦

⎡ − 10.45 116.58 − 209.06 94.41 4.46 − 13.39 ⎤


⎢ − 54.68
⎢ 141.36 − 305.78 193.47 29.64 − 31.62 ⎥⎥
1 ⎢ − 16.74 15.54 − 66.58 62.27 45.79 18.56 ⎥
∴[ A ]−1 = ⎢ ⎥
372 ⎢ 314.34 − 21.94 116.51 − 88.53 13.46 − 40.54 ⎥
⎢ − 16.36 − 41.29 240.08 − 165.91 − 59.89 198.46 ⎥
⎢ ⎥
⎣⎢ 29.64 − 98.58 − 187.78 − 23.06 − 40.17 39.5 ⎦⎥

⎡ − 0.0281 0.3134 − 0.562 0.2538 0.012 − 0.036 ⎤


⎢ − 0.147
⎢ 0.38 − 0.822 0.5201 0.0797 − 0.085 ⎥⎥
⎢ − 0.045 0.0418 − 0.179 0.1674 0.1231 0.0499 ⎥
∴[ A ]−1 =⎢ ⎥
⎢ 0.845 − 0.059 0.3132 − 0.238 0.0362 − 0.109 ⎥
⎢ − 0.044 − 0.111 0.6454 − 0.446 − 0.161 0.5335 ⎥
⎢ ⎥
⎣⎢ 0.0797 − 0.265 − 0.5048 − 0.062 − 0.108 0.1062 ⎦⎥
SHEET (1)
1) Identify the properties of each of the following matrices:

⎡ 0 .5 0 .2 0 ⎤ ⎡8 0⎤ ⎡8 0⎤
[A] = ⎢⎢ 0.2 0.4 − 0.5⎥⎥ [B ] = ⎢⎢− 1 4 ⎥⎥ [C ] = ⎢⎢ 4 3 ⎥⎥
⎢⎣ 0 − 0 .5 1 ⎥⎦ ⎢⎣ 5 0 ⎥⎦ ⎢⎣− 1 7 ⎥⎦

⎡ 0.5 6 0.4 − 0.3⎤


⎡ 2 0 ⎤ ⎡1 0⎤
⎢ 0 −3 2 0 ⎥⎥
[D ] = ⎢⎢ 3 − 2⎥⎥ [E ] = ⎢ [F ] = ⎢⎢ 3 5 ⎥⎥
⎢ 0 0 1 0 ⎥
⎢⎣ 7 − 3⎥⎦ ⎢ ⎥ ⎢⎣ 5 − 6⎥⎦
⎣ 0 0 0 0.2 ⎦

2) Evaluate the following expressions if possible. If not possible, state


the reason.

[C] + [B] [A] + [C] [F]T

2 x [D] + [F] x [E] x [F]T [E] x [F] [A]-1

[B]-1 [B] x [C] – [A]-1 [F]T x [C]

[D]-1 – [C] x [B]

3) Solve the following simultaneous equations:

5X1 – 2X2 + X3 = 9

X1 + 6X2 – 2X3 = 4

X1 + X3 = 1
Chapter (2)
ANALYSIS OF INDETERMINATE STRUCTRES

1.1 Statically Indeterminate Structures

A structure of any type is classified as statically


indeterminate when the number of unknown reaction or internal
forces exceeds the number of equilibrium equations available for
its analysis. Most of structures are statically indeterminate. This
indeterminacy may arise as a result of added support or members,
or by the general form of the structure. For example, reinforced
concrete buildings are almost always statically indeterminate
since the columns and beams are poured as continuous members
through the joints and over supports.

Although the analysis of a statically indeterminate


structure is more involved than that of one that is statically
determinate, there are usually several very important reasons for
choosing this type of structures for design. Most important
reasons for a given loading the maximum stress and deflection
of an indeterminate structure are generally smaller than those of
its statically determinate counterpart. For example, the shown
fixed beam in Fig.(l.l-a), the maximum moment of Mmax= PL/8,
whereas the same beam when simply supported Fig.(l.l-b), will
Fig.(l.l-a) Fig.(l.l-b),

be subjected to twice the moment, that is, Mmax = Pl/4 . As a


result, the fixed support beam has one fourth the deflection and
one half the stress. At its center of the one that is simply-
supported.

Another important reason for selecting a statically


indeterminate structures is that it has a tendency to redistribute
its load to its redundant supports in cases where faulty design or
overloading occurs. In these cases, the structure maintains its
stability and collapse is prevented. This is particularly important
when sudden lateral loads, such as wind or earthquake, are
imposed on the structure. To illustrate, consider again the fixed
beam loaded at its center, Fig.(l-a), As load P is increased, the
beam’s matenal at the fixations and at the center of the beam
begins to yield and forms a localized “Plastic Hinge”, which
causes the beam to deflect. Although the deflection becomes
large, the fixations will develop horizontal force and moment
reactions that will be hold the beam and thus prevent it from
totally collapsing. In the case of the simply supported beam, an
excessive load P will cause the “Plastic Hinge” to form only at
the center of the beam, and due to the large vertical deflection,
the supports will not develop ;the horizontal force and moment
reactions that may be necessary to prevent total collapse.

Although statically indeterminate structures can support a


loading with thinner members and with increased stability
compared to their determinate counterparts, there are cases when
this advantages may instead become disadvantages. The cost
savings in matenal must be compared with the added cost
necessary to fabricate the structure, since often time it becomes
more costly to construct the supports and joints of an
indeterminate structure compared to one that is determinate.
More important, though, because statically lndeterrmante
structures have redundant support reactions, one has to be very
careful to prevent differential settlement of the supports, smce
this effect will introduce internal stress in the structure. For
example, if the fixed support at one end of fixed beam Fig.(l-a)
were to settle, additional moment would be occur. On the other
hand, if the beam was simply supported, then any settlement of
its end would not cause the beam to deform, and therefore no
stress or bending moment would be developed in the beam. In
general, then, any deformation, such as that caused by relative
support displacement, or changes in member lengths caused by
temperature or fabrication errors, will introduce additional
stresses in the structure, which must be acknowledged when
designing indeterminate structures.

1-2 Method of Analyses

When analyzing any indeterminate structures, it is


necessary to satisfy equilibrium, compatibility, and force-
displacement requirements for the structure. Equilibrium is
satisfied when the reactive forces hold the structure at rest, and
compatibility is satisfied when the various segments of the
structure fit together without intentional breaks or overlaps. The
force -displacement requirement depend upon the way the
material responds, which assumed linear-elastic response, hi
general there are two different ways to satisfy these requirements
when analysing a statically indeterminate structure, the force or
displacement method.

1.2.1 Flexibility or Force Method

The force method was originally developed by J.G.


Maxwell and refined by Otto Mohr and Muller-Breslau. This
method was one of the first available for the analysis of statically
indeterminate structures. The force method consists of writing
equations that satisfy the compatibility and forces displacement
requirements for the structure and involve redundant forces as
the unknowns. The coefficients of these unknowns are called
flexibility coefficients. Since compatibility forms the basis for
this method, it has sometimes been referred to as the
compatibility method or the method of consistent deformations.
Once the redundant forces have been determined, the structure
are determined by satisfying the equilibrium requirements for
the structure. The fundamental principles involved in applying
this method are easy to understand and develop.

1.2.2 Stiffness or Displacement Method

The displacement method of analysis is based on first writing


force displacement relations for the members and then satisfying the
equilibrium requirements for the structures. In this case the
unknowns in the equations are displacements and their
coefficients are called stiffness coefficients. Once the
displacements are obtained the forces are determmed from the
compatibility and force-displacement equations. A matrix
formulation of this method will given.

1.2.3 Comparison between the Flexibility and Stiffness Methods

1.2.3.1 Flexibility = deformation per unit force.

δ Cm/t

The deflection (d) of the shown


Spring Fig. (1-2) is; d = δ .P

1.2.3.2. Stiffness:

Stiffness = force per unit deformation

=K (t/cm or t.m/rad.)

For the shown spring in Fig. (2), the force required to


produce a displacement d is;

P=K.d

The two methods can be compared as shown in the ollowing


table.

Flexibility Method Stiffness Method


- Unknown redundants X Unknown displacements d (θ,∆)
- Flexibility matrix 8 Stiffness matrix K

- Displacements due to X = δ .X Force due d = K . d


Force due to loads =Ro
- Displacements due to load = δ o
Equilibrium Ro + K.d = 0
- For Compatibility δo + δ X = 0
Solve for d
- Solve for X
Indeterminate Constraints are Additional restrains are added to
released and the resulting fix all degrees of freedom and the
deformation discontinuities values of these restraints
calculated. These redundant calculated . The restrains are then
actions are then replaced to removed to allow deformations
restore the continuity. and restore equilibrium. The
resulting equilibrium equations
are solved to get displacements
and subsequently the forces are
determined.

The force or flexibility method of analyses discussed in the


previous course of second year civil;. For example the
methods of consistent deformation, three moment equation,
and column analogy methods are considered a flexibility or
force method of analysis whereas the slope deflection, moment
distribution, and stiffness methods are a displacement or
stiffness method of analysis.
Chapter (3)
SLOPE DEFLECTION METHOD
3.1 Introduction:-

The methods of three moment equation, and consistent


deformation method are represent the FORCE METHOD of
structural analysis, The slope deflection method use displace-
ments as unknowns, hence this method is the displacement
method.

In this method, if the slopes at the ends and the relative


displacement of the ends are known, the end moment can be
found in terms of slopes, deflection, stiffness and length of the
members.

In- the slope-deflection method the rotations of the joints


are treated as unknowns. For any one member bounded by two
joints the end moments can be expressed in terms of rotations.
In this method all joints are considered rigid; i.e the angle
between members at the joints are considered not-to change in
value as loads are applied, as shown in fig 1.

joint conditions:- to get θB & θC

MBA+MBC+MBD = 0 …………. (1)

MCB+MCE =0 …………. (2)


Figure (1)

3.2 ASSUMPTIONS IN THE SLOPE DEFLECTION METHOD


This method is based on the following simplified
assumptions.
1- All the joints of the frame are rigid, i.e, the angle between
the members at the joints do not change, when the
members of frame are loaded.
2- Distortion, due to axial and shear stresses, being very
small, are neglected.
3.2.1 Degree of freedom:-
The number of joints rotation and independent joint
translation in a structure is called the degrees of freedom. Two
types for degrees of freedom.
In rotation:-
For beam or frame is equal to Dr.

Dr = j-f
Where:
Dr = degree of freedom.
j = no. of joints including supports.
F = no. of fixed support.
In translation:-
For frame is equal to the number of independent
joint translation which can be give in a frame. Each
joint has two joint translation, the total number or
possible joint translation = 2j. Since on other hand
each fixed or hinged support prevents two of these
translations, and each roller or connecting member
prevent one these translations, the total number of
the available translational restraints is;
2f + 2h + r + m where
f = no. of fixed supports.
h = no. of hinged supports.
r = no. of roller supports.
m = no. of supports.
The degree of freedom in translation, Dt, is given by:-

Dt = 2j-(2f+2h+r+m)

The combined degree of freedom for frame is:-


D = Dr + Dt
= j-f + 2j – (2f + 2h + r + m)
D = 3j – 3j – 2h – r - m

The slop defection method is applicable for beams and


frames. It is useful for the analysis of highly statically
indeterminate structures which have a low degree of kinematical
indeterminacy. For example the frame shown in fig. 2.a

(b)

The frame (a) is nine times statically indeterminate. On other


hand only tow unknown rotations, θb and θc i.e Kinematically
indeterminate to second degree- if the slope deflection is used.
The frame (b) is once indeterminate.

3.3 Sign Conventions:-


Joint rotation & Fixed and moments are considered
positive when occurring in a clockwise direction.

2 M A.L M A.L
θ Al = =
3 2 EI 3 EI

1 M A.L − M A.L
θ BI = = =
3 2 EI 6 EI

1
hence θ B1 = θ
2 A1

1 M B.L − M B.L
θ A2 = =
3 2 EI 6 EI

2 MB.L MB.L
θ B2 = =
3 2 EI 3 EI

θB1 + θB2 = 0
Hence: MA = 2MB

and θA = θA1 -θA2

= M A .L
3 EL − M A .L
12 EL

θA = 3 MA. L
12 EI

4EI
MA = .θA
L
2EI
MB = .θA
L

Relation between Δ & M


Δ
R =
L
by moment area method or
by conjugate beammethod.
Δ = ∑ M at B
M.L 2L
= ( )
4EI 3

M.L2
=
6 EI

6 EI
M = Δ
L2
6EI
= .R
L
R (+ ve) when the rotation of member AB with clockwise.
3.4Fixed and moments:
As given in the chapter of Moment distribution method.
3.5 Derivation of slope deflection equation:-

4 EI
Ma1 = θA
L
2 EI
Mb1 = θA
L
2 EI
Ma2 = θB
L
4 EI
Mb2 = θB
L

Required Mab & Mba in term of (1) θA, θB at joint


B

(2) rotation of member (R)


(3) loads acting on member
First assume:-
Get Mfab & Mfba due to acting loads. These fixed and
moment must be corrected to allow for the end rotations θA,θB
and the member rotation R.
The effect of these rotations will be found separately.
4EI
Ma1 = .θ A
L
2EI
Mb1 = .θ A
L
4EI
Mb2 = .θ B
L
2EI
Ma2 = .θ B
L
− 6 EI
Mb3 = M a3 = .Δ
L2
− 6 EI
= .R
L
by Superposition;
Mab = Mfab + Ma1 + Ma2 + Ma3
4EI 2EI − 6EI
Mf ab + .θ A + θB + .R
L L L
2EI
M ab = Mf ab + (2θ A + θ B − 3R )
L

In case of relative displacement between the ends of members,


equal to zero (R = 0)
2EI
M ab = Mf ab + (2θ a + θb)
L
2 EI
M ba = Mf ba + (2θ b + θ A )
L

2 EI
The term ( ) represents the relative stiffness of member say
L
(K) hence:
M ab = Mf ab + K ab (2θ A + θ b )
M ba = Mf ba + K ba (2θ B + θ a )

Note:
Δ
= R is (+ ve) If the rotation of member with clockwise.
L
And (– ve) If anti clockwise.
− 6 EI
M = .Δ ( with + ve R )
L2
− 6 EI
M = .Δ ( with − ve R )
L2

3-5-1 Example 1
Draw B.M.D. S.F.D
Solution:-
1 2.66
1- Relative stiffness:- KAB : KBC = : 1: 2
6 8
2- Fixed and Moment:-
3 × 62
MFBA = = − 9 t.m.
12

3 × 62 3 × 82
MFBA = + =+ 9 , MFBC = + = − 18
12 12

3 × 82
MFCB = + = + 18
12
3- Two unknown θB + θC then two static equations are
required. 1) ∑ MB = 0
2) MC = 0
Hence:
MBA + MBC = 0 ……………… (1)
MBC = 0………………. (2)

But:
MAB = - 9 + θB
MBA = 9 + 1 (2θB)
MBC = -16 + 2 (2θB + θC)
MCB = +16 + 2 (2θC + θB) B

From eqns. (1&2)


9 + 2θB + (- 16 + 2 (2θB + θC) = 0
B B

6θB + 2θC
B =7 …….(3)
and 4θC + 2θB B = - 16
2θC + θB =-8 …….. (4)
from 3 & 4
5 θB B = 15
15
θB = = 3.0
5
θC = - 5.5
1.e. MAB = - 9 + 3.4 = 5.6 t.m
MBA = 9 + 2 × 3.4 = 15.8 t.m

MBC = - 18 + 2 (2 × 3.4) + (- 5.5) = - 15.0 t.m

MCB = 16 + 2 (2.3 – 5.7 + 3.4) = 0.0 (0.k)


1- Unknowns θA , θB , & θC
2- Fixed end Moment
2 × 62
MFAB = MFBC = MFCD = = - 6 t.m … etc
12

3- Condition eqns.
MAB = - 4 t.m, MBA + MBC = 0, & MCB + MCD = 0
4- Slope deflection equations
MAB = - 6 + (2θA + θB) = - 4 B

2θA + θB B =2 ……….(1)
MBA + MBC =0
+ 6 + (2θB + θA) –6 + (2θB + θC) = 0
4θB + θA + θC =0 ……… (2)
MCB + MCD =0
= 6 + 2θC + θB – 6 + 2θC = 0
B

4 θC + θB B =0 ……….(3)

From eqn.3 θC = - θB
4
Substitute in eqn. (2)
Hence: 3.75 θB + θA = 0 B ................(2)
0.5θB + θA
B =1 ………… (2)
3.25θB B =-1
θBB =-1
θA = 1.15
θC = 0.077
Hence:
MAB = - 6 + 2xl.15 + (- .307)
= - 4 t.m 0.K
MBA = 6 + 2x (- .307) + 1.15 = 6.536 t.m
MCB = 6 + 2x .77 + (- .307) = 5.85 t.m
MDC = 6 + .077 = 6.077 t.m

Solution:-
1- Unknown displacements are θB & θD B

2- Equations of equilibrium are:-


MDB = 0 ……….(1)
MBA + MBD + MBC = 0 ………(2)
3- Relative Stiffness:-
KAB: KBC: KBD = 35:31. 5:22 ; 51. 56:1. 4:1.0.
4- Fixed and Moments:
− 9 × 6 × 3× 3
MFAB = = − 6 t.m
9×9

9 × 6 × 3× 6
MFBA = =12 t.m
9×9

− 3× 7 2
MFBD = = − 12.25 t.m
12

− 3× 7 2
MFDB = = − 12.25 t.m
12
From the equations 1 & 2 hence;
MDB = MFDB + (2θD + θB) B

= 12.25 + 1 (2θD + θB) B =0


2θD + θB + 12.25 B = 0 ------- (3)
and MBA = 12 + 1.56 (2θB) B

MBD = 12.25 + 1.0 (2θB + θD) B

MBC = 0 + 1.4 (2θB + 0) B

i.e.
12+1.56 (2θB) – 12.25 + 2θB + θD + 1.4 (2θB) = o
B B B

7.92θB + θD – .25
B = 0 ------------- (4)
0.5θB + θD + 6.125
B = 0 ------------- (3)
i.e 7.42 θB B - 6.375 =o
θBB = 0.86
θD = - 6.55
Hence:
MBA = 12 + 1.56 (2× .86) = 14.68 t.m
MBD = - 12.25 + (2 × .86 – 6.55 × 1) = - 17.08
MBC = 1.4 (2 × .86) = 2.41
MDB = 12.25 + (2 × -6.55) = zero
1
MCB = MBC = 1.205
2
MAB = -6 + 1.56 (.86) = - 4.66
Two equilibrium eqns.
MAB + MAA = 0 ………………… (1)
MBB + MBA + 4= 0 ………………….(2)

Slope deflection eqns.


MAB = o + 1.6 (2θA + θB) B

− 10 ×16
MAA = + (2θ + θ )
A A
8
MAA = - 20 + θA
MBA = o + 1.6 (2θB + θA) B

MBB = - 42.67 + (2θB + θB) B B

= - 42.67 + θB B

Hence:
3.2θA + 1.6θB + θA – 20 = o
B

4.2θA + 1.6θB B = 20 ……………… (1)


- 42.67 + 4.2 θB + 1.6θA + 4 = 0
B

1.6θA + 4.2θB B = 38.67 ……………….(2)


1.6θA + 0.61θB B = 7.62 ……………….(1)

3.59θB B = 31.05
θB B = 8.65
θA = 1.46
MAB = -18.52
MBA = 30
MBB = - 34

Example 5
Draw B.M.D for the shown frame
Solution:-
- Two condition equations.
MAA + MAB =0 …….. …………...(1)
MBA + MBB + 8 = 0 …………………..(2)
1 1
- Relative stiffness : = 1:1.6
16 10
- Slope deflection equations:
MAA = (2θA – θA) = θA
MAB = (2θA – θB) × 1.6 B

MBA = (2θB – θA) × θA


B

MBB = 42.67 + (2θB - θB) B B

Hence:
θA + 3.2θA + 1.6VB = 0 B

4.2θA + 1.6θB B = 0 .… (1)


3.2θB + 1.6θA + θB – 42.67 + 8 = 0
B B

4.2θB + 1.6θA
B = 34.67… (2)
By Solving 1 & 2 θA = - 3.68 , θB = 9.66
B

Hence MAA = - 3.68 , MAB = 3.68 t.m


MBA = 25 MBB = 33

Example 6:
- Draw B.M.D for the given structure.

Solution:- once statically indeterminate.


1- Fixed end moments
8 × 20
MFAB = - = − 20 t.m
8
8 × 20
MFBA = - = − 20 t.m
8
4 × 10
MFBC = - = − 5 t.m
8
10 × 8
MFCB = - = −10 t.m
8
MFDB = 10 t.m

2- From Static:- ∑ MB = o
B
MBA + MBC + MBD = 0
MBA = MFBA + (2θB) B

MBA = 20 + 2θB B ……………….. (1)


MBC = - 5 + 2θB B ……………….. (2)
MBD = - 10 + 2θB B ………………. (3)

Hence: 5 + 6θB = o
B

θBB = - o.833
Hence:
MBA = 18.34 t.m , MBC = -6.67, MBD = -11.67 t.m
MAB = - 20 = - 20.833 t.m
MCB = 5 + θB = - 4.167 t.m
MDB = 10 + θB = 9.167 t.m

Example 7:
Draw B.M.D for the shown frame
Solution:
“ 3 time statically ind.” θA , θB , & θC
B

1- Fixed end moments:


MFAB = - 10
MFBA = + 10
MFBC = - 25

MFCD = MFDC = zero

2- Relative Stiffness 1:1:1


MAB =0 …………………….(1)
MBA + MBC = 0 …………………….(2)
MCB + MCD = 0 …………………….(3)
Equs.
MAB = - 10 + (2θA + θB) B

MBA = 10 + (2θB + θA)


B

MBC = - 25 + 2θB + θC
B

MCB = 25 + 2θC + θB B

MCD = 2θC
MDC = θC
From 1,2 & 3
2θA + θB B = 10 …………….(1)
4θB + θA + θC = 15
B …………….(2)
4θC + θBB = - 25 ……………(3)
By solving the three eqns. hence;
θA = 2.5 θB = 5
B θC = - 7.5
Substitute in eqns of moments hence;
MAB = - 10 + 5 = zero (o.k)
MBA = 10 + 10 + 2.5 = 22.5 t.m
MBC = - 25 +10 – 7.5 = - 22.5 t.m
MCB = 25 – 15 + 5 = 15 t.m
MCD = - 15 t.m
MDC = - 7.5 t.m

3-6 Frames with Translation


Examples to frames with a single degree of freedom in
translation.
Example 8:
Draw B.M.D for the shown frame.
1- Unknowns: θB , θC , Δ

2- Relative stiffness
KAB : KBA : KCD
1 2 1.5
: :
4 8 6
1: 1 : 1
3- Fixed end moments
MFAB = o MBA = o
MFBC = MCB = zero
MFCD = - 6 t.m
MFDC = + 6 t.m
4- From Statics the equilibrium eqns
MBA + MBC = 0 …………………(1)
MCB + MCD = 0 ………………….(2)

5- Shear equation (In direction of X, ∑ × = o)

6 + XA + XD – 8 =o
M BA + M AB M + M DC
6+ + CD − 4 = 0 (3)
4 6

MBA + MAB MCD + MDC


hence XA + and xD = +4
4 6
6- Slope deflection eqns:
Δ Δ
MBA – 0 + 1 (2θB – 3 ), MAB = 0 + 1 (θB – 3. )
4 4
B B
MBC = 0 + 1 (2θB + θC) B

Hence: 4θB – 0.75Δ + θC = 0


B (1)
MCB = 0 + 1 (2θC + θB) B

Δ
MCD = - 6 + 1 (2θC – 3 ),
6
Δ
MDC = + 6 + 1 (θC-3 )
4
Hence:
1
4 θC + θB - Δ =6 (2)
2
B

(2θ B − .75 Δ ) + (1θ B − .75 Δ ) (− 6 + 2θ C − Δ ) + (6 + 1θ C − Δ


2+ + =0
4 6
1
2 + 0.75 θB - .375 Δ + θC – 0.1667Δ = o
2
B

θB + .67 θC - 072Δ = - 2.66


B (3)
Subtract (3) from (2)
1
3.33 θ + 0.288 Δ = 8.33
2

θC – 0.067 Δ = 2.6 (4)


Subtract (1) from (2) × (4)
15 θC – 1.25 Δ = 24
θC – 0.08 Δ = 1.6 (5)
From (4) & (5) 0.147Δ = 1
Δ = 6.80
θC = 2.149
θB = 0.799
B
MBA = - 3.5 t.m ,MAB = - 4.301 t,m, MBC = 3.79
MCB = 5.1 t.m ,MCD = - 5.1 t.m , MDC = 4.744
Example 9:-
Write the shear & condition eqns for the following frame.
Solution:-
Three unknowns: θB , θC, Δ

Condition equations:
MBA + MBC = o (1)
MCB + MCD = o (2)
Shear eqn.
XA +XB + P1 + P2 = o
B

P1 M AB + M BA M + M DC
(− + ) + ( CD ) P1 + P2) = o (3)
2 h1 h2
Example 10:
Find the B.M.D for the shown structure.
Solution:-
θD = θE = o
θC = - θC
θB = - θB
B B

1- Unknown displacements are: θB , θC , Δ


2- Relative Stiffness:
AB : BE : BC : CD : ED
1 2 1 1 1
: : : :
5 3 5 3 3
3 : 10: 3 : 5: 5

3- Fixed end moment:-


4 × 36
MFBE = - = − 12 t.m
12
MFEB = + 12 t.m
1.5 × 36
MFCD = = − 4.5 t.m
12
MFDC = + 4.5
4- Equilibrium equations:-
1- MCD + MCB = o
2- MBC + MBA + MBE = o
M CD + M DC M DE + M ED
3- Shear condition:(33–16.5)+ +
6 6
MCD = - 4.5 + 5 (2θC + θD – 3R)
MCB = 0 3 (2θC + θB) B

MBC = 0 + 3 (2θB + θC) B

MBA = 0 + 3 (2θB) B

MBE = - 12 + 10 (2θB – 3R) B

Hence
- 4.5 + 10 θC – 15 R + 6 θC + 3 θB = 0 B

16θC + 3θB – 15R – 4.5 = 0


B (1)
And
16θB + 3θC + 6θB – 12 + θB - 3θR = 0
B B B

3θC + 32θB – 30R – 12 = 0


B (2)
and
15θ C − 30 R 30 θ B − 60 R
16.5( + )=0
6 6
2.5 θC + 5θC + 17R + 16.5 =0 (3)
by solving equation 1,2 & 3 get
MAB = + 6.66 t.m
MBA = + 13.32 t.m
MBC = + 19.0 t.m
MCB = + 18 t.m
MBE = - 32.32 t.m
MEB = - 30.53 t.m
MCD = - 18 t.m
MDC = - 18.43 t.m

3-7 Frame with multiple degree of freedom in translation.


Example 11:
Write the shown equations and condition eqns for the given
frame.
Solution
Unknowns: θB , θC , θD , θE , ∆1 , ∆1
Condition eqns
MBE + MBA + MBC =0 (1)
MCB + MCD =0 (2)
MDC + MDE =0 (3)
MEB + MEF + MED = 0 (4)
Shear eqns :
Equilibrium of the two stories.
At sec (1) – (1) :-
(Level CD)
P2 + Xc + XE =0
M CB + M BC M + M ED
P2 + + DE =0
h2 h2
At sec. (2) – (2):-
(Level BE) or ∑ x = 0
P 1 + P2 + x A + xF = 0
M BA + M AB M + M FE
P 1 + P2 + + EF =0
h1 h1
Example 12:-
Draw B.M.D for the given structure.

S
olution:-
1- Relative Stiffness:-
2- Equilibrium equations:-
MAB + MAC =0 (1)
MBA + MBD =0 (2)
MCA + MCD + MCE = 0 (3)
MDB + MDF + MDC = 0 (4)

Σx = 0 at Level A-B
M AC + M CA M + M DB
2 + (6-3) + + BD = 0 (5)
6 6
Σx = 0 at Level CD
M CE + M EC M + M FD
11 + + DF = 0 (6)
6 6
MAB = - 8 + 1 (2θA + θB) B

MAC = 3 + (2θA + θC – 3R1)


MAC = - 3 + (2θC + θA – 3R1)
MCA = 16 + (2θB + θA)B

MBD = 0 + (2θB + θD – 3R1)


B

MDB = 0 + (2θD + θB - 3R1)


B

MDF = 0 + (2θD + 0 - 3R2)


MFD = 0 + (θD - 3R2)
MCD = - 48 + 2 (2θC + θC)
MCD = + 48 + 2 (2θD + θC)
MCE = - 8 + (2θC - 3R2)
MEC = + (θC - 3R2)
3- Fixed end moment:-
9× 4×8 × 4
MFAB = − = − 8 t.m
12 ×12

9×82 × 4
MFBA= − = + 16 t.m
12 2

1× 6 2
MFAC = = + 3 t.m
12
1× 6 2
MFCA= = − 3 t.m
12

4 ×12 2
MFCD = = − 48 t.m
12
MFDC = + 48 t.m
4- Unknown displacement:
θA , θB , θC , θD , ∆1 , ∆2
by Solving the six equations one can get;
MAB = - 3.84 t.m
MBA = + 18.39 t.m
MAC = 3.84 t.m
MCA = + 7.29 t.m
MBD = - 18.39 t.m
MDB = - 22.97 t.m
MCD = - 11.15 t.m
MDC = - 53.44 t.m
MCE = 3.87 t.m
MEC = - 13.44 t.m
MDF = - 30.47 t.m
MFD = - 26.15 t.m
Example (13):-
Write the shear equations & equilibrium equations for the shown
frame.
Solution:
Shear eqns:
XCE + XBA + P1 = 0 ……………………....(1)
M EC + M CE M + M BA
+ AB + P1 = o
h1 h1 + h2

xD + xG + xE + P2 = 0 --- (2)
M DE + M ED M GF + M FG M EC + M CE
+ − + P2 = 0
h2 h2 h1

Or:
XA + XD + XG + P1 + P2 =0
M AB M + M ED M + M FG
+ DE + GF + P 1 + P2 = 0
h1 + h2 h2 h2
Example 14:-
a- Write the equations of equilibrium including the shear
equations for the frame.
b- Write the slope deflection equations in matrix for
members CE & GH.
c- By using the slope - deflection method; sketch elastic
curve.
d- Sketch your expected B.M.D

Solution:-
(Unknowns = θC , θD , θE , θF , θG , θA+θK , θL , ∆1 ,
∆2 , ∆3 , ∆4
Relative stiffness: 1 : 1
a- equilibrium equations
MKL + MKG =0 (1)
MLK + MLH =0 (2)
MGK + MGH + MGE = 0 (3)
MHG + MHL + MHF = 0 (4)
MEG + MEC + MFF = 0 (5)
MFE + MFD + MFH = 0 (6)
MCE + MCD + MCA = 0 (7)
MDC + MDB + MDF = 0 (8)

Shear equations:-
a- at Level GH
5 + 10 + (XG – 5) + XH = 0 (9)
Where:
M GK + M KG
XG =
5
M HL + M LH
XH =
5
b- at Level EF
5 + 10 + 20 + (XE – 5) + XF =0
30 + XE + XF =0 (10)
Where:
M EG + M GE
XE =
5
M FH + M HF
XF =
5
c- at Level CD
5 + 10 + 10 + 30 + (XC – 5) + XD =0
50 + XC + XD = 0 ………(11)
Where:
M CE + M EC
XC =
5
M DF + M FD
XD =
5
d- at Sec AB:-
5 + 10 + 10 + 10 + 40 + (XA – 5) + XB
B =0
70 + XA + XB B = 0 …..(12)
M AC + M CA
XA =
5
M CE + M EC
XB =
5
B

3-8 Slope deflection eqns in matrix form:


1- Member CE
2EI Δ 2 − Δ1
MCD = MFCE + (2θC + θE –3 )
5 5
2EI Δ 2 − Δ1
MEC = MFEC + (2θE + θC –3 )
5 5
Where:
2× 52
MFCE = - = - 4.16 t.m
12
MFEC = + 4.16 t.m
In Matrix form:
MCE - 4.16 2 1–3 θC
2EI
= + θE
5
MEC 4.16 1 2–3 R2

Where:
Δ 2 − Δ1
R2 =
5
2- member GH
MGH - 16.67 2 1 θG
26I
= +
5
MHG + 16.67 1 2 θH

d- B.M.D

Example 15:-
By using slope deflection method;
1- Draw B.M.D for the shown frame.
2- Sketch elastic curve.
Solution:
1- Relative stiffness 1: 1
2- unknowns: θB = - θB (From symmetry)
B B

3- Equilibrium eqns
MBA + MBC + MBD + MBB = o (1)
4-Fixed end moments
4× 62
MFAB = = - 12 t.m
12
MFBA = = + 12
MFBC = MFCB = MFBD = MFDE = o
2 ×12 2 8 ×12
MFBB = - − = - 36 t.m
12 8

4- Slope deflection eqns


MAB = - 12 +(θB)
MBA = 12 + 2θB
MBC = 2θB
MBD = 2θB
MBB = - 36 + θB
MCB = θB
MDB = θB

From eqn (1)


(12 + 2θB) + (2θB) + (2θB) + (- 36 + θB)
B B B B =0
7θB - 24
B =0
θB B = 3.4286
hence
MAB = - 8.57 t.m
MBA = 18.86 t.m
MBC = 6.86 t.m
MBB = - 32.58 t.m
MCB = 3.428 t.m
MDB = 3.428 t.m
MBD = 6.86
The Free Body Diagram to find the S. F. & N. F.
SHEET (3)

1) Draw S.F.D. and B.M.D. for the statically indeterminate


beams shown in figs. From 1 to 10.
2) Draw N.F.D., S.F.D. for the statically indeterminate
frames shown in figs. 11 to 17. Using matrix approach 1.
THE STIFNESS MATRIX METHOD:
APROACH 1 ( SLOP DEFLECTION IN MATRIX FORM )
In this method, the slop deflection equations method is formulated using matrix operations.
The method can be formulated for weather the members have relative displacements or not

Members without relative displacements

In this approach, the slop deflection equations for each member are collected in a
matrix form. For example, the slop deflection equations for member AB in a matrix form
become

⎡ 4 EI 2 EI ⎤
⎡M AB ⎤ ⎡M FAB ⎤ ⎢ L L ⎥ ⎡θ AB ⎤
⎢M ⎥ = ⎢M ⎥ + ⎢ 2 EI 2 EI ⎥ ⎢⎣θ BA ⎥⎦
(4.108)
⎣ BA ⎦ ⎣ FBA ⎦ ⎢ ⎥
⎣ L L ⎦ AB

In a short form, one may write equation 4.108 as

MAB = MFBA + SAB DAB (4.109)

For a structure consists of a set members, Equation 4.109 can be repeated


For each member. The structure shown in Figure 4.76, for example, the slope
Deflection equation are collected in a matrix form as

B C E

A D

Figure 4.76

⎡ M AB ⎤ ⎡ M FAB ⎤ ⎡ S AB 0 0 0 0 ⎤ ⎡ D AB ⎤
⎢M ⎥ ⎢M ⎥ ⎢ 0 S BC 0 0 0 ⎥⎥ ⎢⎢ D BC ⎥
⎢ BC ⎥ ⎢ FBC ⎥ ⎢ ⎥
⎢ M AC ⎥ = ⎢ M FAC ⎥+⎢ 0 0 S AC 0 0 ⎥ ⎢ D AC ⎥ (4.110)
⎢ ⎥ ⎢ ⎥ ⎢ ⎥⎢ ⎥
⎢ M CD ⎥ ⎢ M FCD ⎥ ⎢ 0 0 0 S CD 0 ⎥ ⎢ D CD ⎥
⎢⎣ M CD ⎥⎦ ⎢⎣ M FCD ⎥⎦ ⎢⎣ 0 0 0 0 S CE ⎥⎦ ⎢⎣ D CD ⎥⎦

Equation 4.110 can be written for any structure whose members are
not subjected to relative displacements. It can be expressed in a general form as
M m = M Fm + [S m ]D m (4.111)
In which M m member end moments, MFM contains the members fixed end moments
And [Sm] is the augmented member's stiffness matrices

The compatibility condition for the frame of figure 4.76 can be stated as

Follows:

θ AB = θ AC = θ A

θ BA = θ BC = θ B
θCA =θCB =θCD =θCE =θC
θ DC = θ D (4.112)
Where θA , θB , θC ,and θD are the cinematic variables at the joints of this structure .
Equation 4.112 can be expressed in a matrix form as

⎡θ AB ⎤ ⎡1 0 0 0⎤
⎢θ ⎥ ⎢
⎢ BA ⎥ ⎢0 1 0 0⎥⎥
⎢θ BC ⎥ ⎢0 1 0 0⎥
⎢ ⎥ ⎢ ⎥
⎢ θ CB ⎥ ⎢0 0 1 0⎥ ⎡θ A ⎤
⎢θ ⎥ ⎢1 ⎢ ⎥
0 0 0⎥ ⎢θ B ⎥
⎢ AC ⎥ = ⎢ ⎥
⎢θ CA ⎥ ⎢0 0 1 0⎥ ⎢θ C ⎥ (4.113)
⎢θ ⎥ ⎢0 ⎢ ⎥
0 0 1⎥ ⎣θ D ⎦
⎢ DC ⎥ ⎢ ⎥
⎢θ CD ⎥ ⎢0 0 1 0⎥
⎢ ⎥ ⎢
⎢θ CE ⎥ ⎢0 0 1 0⎥

⎢⎣θ EC ⎥⎦ ⎢⎣0 0 0 0⎥⎦

Equation 4.113 can be expressed in a short form by

Dm = C D ( 4.114 )
Where C is called the compatibility or connectivity matrix.
In order to solve the problem one has to apply the equilibrium conditions at the
free joints. For the structure of figer 4.76 one has the following equilibrium condition:
MA=MAB + MAC
MB=MBC + MBA
MC=MCB + MCD+MCD+MCE
MD=MDC

Equation 4.115 can also be expressed in a matrix form as


⎡M AB ⎤
⎢M ⎥
⎢ BA ⎥
⎢M BC ⎥
⎢ ⎥
⎡M A ⎤ ⎡1 0 0 0 1 0 0 0 0 0⎤ ⎢M CB ⎥
⎢ ⎥ ⎢
⎢ M B ⎥ = ⎢0 1 1 0 0 0 0 0 0 0⎥⎥ ⎢M AC ⎥
⎢ ⎥ (4.116)
⎢ M C ⎥ ⎢0 0 0 1 0 1 0 1 1 0⎥ ⎢M CA ⎥
⎢ ⎥ ⎢ ⎥
⎣ M D ⎦ ⎣0 0 0 0 0 0 1 0 0 0⎦ ⎢M DC ⎥
⎢ ⎥
⎢M CD ⎥
⎢ ⎥
⎢M CE ⎥
⎢⎣M EC ⎥⎦

Comparing equation 4.116 with equation 4.113,


One may write equation 4.116

M=CT Mm (4.117)
Where M is a vector which contains the external moments at the free joints
Substituting equation 4.111 and 4.114 into equation 4.117 one obtains

M=CT MFm+CT[Sm] C D (4.118)

Equation 4.118 can also be written in a shorter form as

M=MF + [S] D (4.119)

Where MF and [S] are, respectively, obtained from

MF =CT MFm (4.120)

[S] = CT [Sm] C (4.121)

Equation 4.119 is now in a suitable form to be solved for the unknown displacements D as follows

D = [S]-1(M-MF ) (4.122)

It is a common practice to call the terms (-MF) in equation 4.122 by the equivalent joint moment due
to the direct loadings on the members. It is obvious that this matrix approach is suitable for computer
applications. The analyst stories the structural data in the form of [Sm], the loading data in the form
of M and MFm, and finally the compatibility matrix C which is obtained from boundary and
connectivity conditions. The matrices MF and S are then calculated by matrix multiplication
according to equations 4.120 and 4.121. The free displacements D are obtained according to
equation 4.122. The member end moments are determined from equation 4.111 using equation 4.114

Example 4.26
Determined the bending moment diagram for the beam shown in figure 4.77 using the stiffness
matrix method – abrosh I (EI=105 KN.m2 ,α=10-5/ οc )
50KN

10KN/m
A B C

Figure 4.77

Solution
The stiffness matrices SAB and SBC, are determined as follows:

⎡ 4 EI 2 EI ⎤
⎢ 5 ⎥ = EI ⎡0.80 0.40⎤
S AB = S BC =⎢ 5
2 EI 4 EI ⎥ ⎢0.40 0.80⎥
⎣ ⎦
⎢ ⎥
⎣ 5 5 ⎦

The members stiffness matrix [Sm] is composed as follows:

⎡0.8 0.4 0 0⎤
⎢0.4 0.8 0 0 ⎥⎥
[S m ] = EI ⎢
⎢0 0 0.8 0.4⎥
⎢ ⎥
⎣0 0 0.4 0.8⎦

The degree of freedom in this structure is two, which represents the angles of
Rotation at B and C . The compatibility equations are put in a matrix form as

⎡θ AB ⎤ ⎡0 0⎤
⎢θ ⎥ ⎢
⎢ BA ⎥ = ⎢1 0⎥⎥ ⎡θ B ⎤
= CD
⎢θ BC ⎥ ⎢1 0⎥ ⎢⎣θ C ⎥⎦
⎢ ⎥ ⎢ ⎥
⎣θ CB ⎦ ⎣0 1⎦
Figure 4.78

The fixed end moments are determined for the cases shown in Figure 4.78 using the
Tables and according to the signs of the slope deflection equation as follows:

4 EI 50 * 5
M FAB = θA + = 191.25 KN .m
5 8

2 EI 50 * 5
M FBA = θA − = 48.75 KN .m
5 8

T1 − T2
M FBC = EIα = 50 KN .m
h

M FBC = −50 KN.m


The fixed end moments are collected to form matrix MFm as follows:
T
M Fm = [191.25 48.75 50 -50]
The matrix MF and [S] are obtained according to equations 4.120 and 4.121 as
Follows:

T ⎡0 1 1 0 ⎤ ⎡98.75⎤
M F = C M Fm = ⎢ ⎥ M Fm = ⎢ ⎥
⎣0 0 0 1 ⎦ ⎣− 50 ⎦

0.4 ⎤
[S ] = C T [S m ]C = EI ⎡⎢
1.6
⎣ 0 .4 0 . 8 ⎥⎦

Substituting into the equilibrium equation one obtains

M=MF + [S] D
⎡M B ⎤ ⎡98.75⎤ ⎡1.6 0.4⎤ ⎡θ B ⎤ ⎡0 ⎤
=
⎢M ⎥ ⎢− 50 ⎥ + E ⎢0.4 0.8⎥ ⎢θ ⎥ = ⎢− 20⎥
⎣ C⎦ ⎣ ⎦ ⎣ ⎦⎣ C ⎦ ⎣ ⎦

The deformation D is solved to have

⎡θ B ⎤ 1 ⎡− 81.25⎤
D=⎢ ⎥ ⎢ ⎥
⎣θ C ⎦ EI ⎣78.125 ⎦

The members end moments are calculated from equation 4.111as follow:

M m = M Fm + [S m ]D m

⎡0 ⎤
⎡M AB ⎤ ⎡191.25⎤ ⎡0.8 0.4 0 0 ⎤⎢ ⎥ ⎡158.75 ⎤
⎢M ⎥ ⎢ − 81.25
⎥ ⎢
⎢ BA ⎥ = ⎢48.75 ⎥ + EI ⎢0.4 0.8 0 0 ⎥⎥ ⎢ EI ⎥ ⎢− 16.25⎥
⎢ ⎥=⎢ ⎥ KN .m
⎢M BC ⎥ ⎢50 ⎥ ⎢0 ⎥ − 81.25 ⎢16.25 ⎥
EI ⎥ ⎢
0 0.8 0.4 ⎢
⎢ ⎥ ⎢ ⎥ ⎢ ⎥⎢ ⎥
⎣M CB ⎦ ⎣− 50 ⎦ ⎣0 0 0.4 0.8⎦ 78.125 ⎥ ⎣− 20 ⎦
⎢ ⎥
⎣ EI ⎦

Which gives the same results obtained previously. The bending moment diagram is
Given again Figure 4.79

Example 4.27

Determine the bending moment diagram for the frame shown in Figure 4.80 using the
Stiffness matrix method approach I, where Support B has displaced down 1cm, and
EI= 105 KN.m2

Solution

The stiffness matrices SAC and SCB are determined as follows:

1 5 8 .7 5 K N .m

2 5 K N .m 2 0 K N .m
1 6 .2 5 K N .m

Figure 4.79
20KN

C B

50KN

A
Figure 4.80

⎡ 4 EI 2 EI ⎤ ⎡ 4 EI 2 EI ⎤
S AC

=⎢ 6 6 ⎥, S CB

=⎢ 8 8 ⎥
2 EI 4 EI ⎥ 2 EI 4 EI ⎥
⎢ ⎥ ⎢ ⎥
⎣ 6 6 ⎦ ⎣ 8 8 ⎦

The member's stiffness matrix [Sm] is obtained form

⎡0.667 0.333 0 0 ⎤
⎢ 0 ⎥⎥
[S m ] = ⎡⎢
S AC 0 ⎤ ⎢0.333 0.667 0
= EI
⎣ 0 S CB ⎥⎦ ⎢ 0 0 0.5 0.25⎥
⎢ ⎥
⎣ 0 0 0.25 0.5 ⎦

The degree of freedom in this structure is two, which represents θC and θB .


B

The compatibility equations are θAC =0, θCA = θC, and θBC = θB . These relations are
Put in a matrix form as follows:

⎡θ AB ⎤ ⎡0 0⎤
⎢θ ⎥ ⎢
⎢ CA ⎥ = ⎢1 0⎥⎥ ⎡θ C ⎤
⎢θ CB ⎥ ⎢1 0⎥ ⎢θ ⎥ = C D
⎣ B⎦
⎢ ⎥ ⎢ ⎥
⎣θ BC ⎦ ⎣0 1⎦

The fixed end moments are determined for each member using the tables for the cases
Shown in Figure 4.81, and in the singes of the slope deflection equation.

50 * 6
M FAC = = 37.5 KN.m
8
MFAC = -37.5 KN.m
50KN

C B
C

50 KN

A
Figure 4.81

20 * 8 6EI ⎛ 1 ⎞
M FCB = + + 2 ⎜ ⎟ = 113.75 KN.m ; M FBC = −20 + 9375 = 73.75 KN.m
8 8 ⎝ 100 ⎠

The calculations of MF and S follow equations 4.120 and 4.121 as follows:

= [37.5 − 37.5 113.75 73.75]


T
M FM

T ⎡0 1 1 0⎤ ⎡76.25⎤
M F = C M FM = ⎢ ⎥ M FM = ⎢ ⎥ KN .m
⎣0 0 0 1 ⎦ ⎣73.75⎦

⎡ 0.333 0 ⎤
⎢0.667 0 ⎥⎥ ⎡1.167 0.25⎤
[S] = C T [S m ] C = EI⎢ = EI ⎢ ⎥
⎢ 0.5 0.25⎥ ⎣ 0.25 0.5 ⎦
⎢ ⎥
⎣ 0.25 0.5 ⎦

Substituting into Equation 4.119, one obtains

⎡M C ⎤ ⎡76.25⎤ ⎡1.167 0.25⎤ ⎡θ C ⎤ ⎡0⎤


⎢M ⎥ = ⎢73.75⎥ + EI⎢ 0.25 0.5 ⎥ ⎢θ ⎥ = ⎢0⎥
⎣ B⎦ ⎣ ⎦ ⎣ ⎦⎣ B ⎦ ⎣ ⎦

The solution for θC and θB gives

⎡θ C ⎤ 1 ⎡− 37.788 ⎤
⎢θ ⎥ = ⎢ ⎥ rad
⎣ B ⎦ EI ⎣− 128.606⎦

The end moments are now calculated using Equation 4.111 as follows:

M m = M FM + [S m ] D m
⎡0 ⎤
⎡M AC ⎤ ⎡37.5 ⎤ ⎡0.667 0.333 0 0 ⎤⎢ ⎥ ⎡24.916 ⎤
⎢M ⎥ ⎢ − 37.788
⎥ ⎢ ⎥
0 ⎥⎢ EI ⎥ ⎢− 62.704⎥
⎢ CA ⎥ = ⎢− 37.5 ⎥ + EI ⎢ 0.333 0.667 0
⎢ ⎥=⎢ ⎥ KN.m
⎢M CB ⎥ ⎢113.75⎥ ⎢ 0 0 0.5 0.25⎥ ⎢− 37.788 ⎥ ⎢62.704 ⎥
⎢ ⎥ ⎢ EI ⎥
M 73.75
⎥ ⎢
0 0 0.25 0.5
⎥⎢ ⎥ ⎢0
⎣ BC ⎦ ⎣ ⎦ ⎣ ⎦ ⎢− 128.606 ⎥ ⎣ ⎦
⎣ EI⎦

The bending moment diagram is given in Figure 4.82, which is the same as the
Results of Example 4.7.

BMD
Figure 4.82

4.6.2 Effect of Members with Moments Releases


if the moment in a member is released at one end, one may use the modified
Slope deflection equation, given in Equation 4.15 For any member AB which has a
hinge at B, one may write Equation 4.15 in a matrix form as follows:
SHEET (4)
Draw the bending moment diagram for the following structures using the matrix approach 1
method (EI=104 t.m2 ,α=10-5/ οc )
CHAPTER 3
Stiffness Matrix Method
3-1- DEFINITION
The stiffness method is a method of analysis, where the main unknowns
are the displacements of joints. These unknowns are determined from
equilibrium. The method can be used for determination of displacements
and internal forces due to
• external loads,
• environmental changes (temperature and shrinkage), and
• support movement.

The stiffness method is applicable to skeletal structures (beams, plane and


space frames and trusses, and grids), and continuum structures (plates,
shells, and three dimensional solids).
(Question: what is the difference between skeletal and non-skeletal
structures (continuum structures)?)

The steps of this method are:

1- Modeling
• Model the structure with a number of members and
joints,
• Define types of members and connections,
• Identify the relevant unknown components of joint displacements.

2- Load vector
• For each member determine the member fixed end
forces (which are the reactions due to loads if all the
relevant joint displacements were prevented).

3- Stiffness matrix
• For each member determine the member end forces
due to joint displacements (i.e. member stiffness
equations).

4- Equilibrium equations
• Write the equations of equilibrium of all joints (under
the forces in steps 2 and 3, and joint loads).

5- Solving equilibrium equation


• Solve equilibrium eq. to determine joints
displacements.

6- Internal forces
• Use the resulting joint displacements to determine the
total member end forces.
The above steps can be illustrated by the following example:

EXAMPLE
Determine the displacements 3t
and member end forces in the 2t/m
shown frame due to the
given loads.
3.00

2.00 6.00

STEP 1: Modeling.
D2

D3 D1 D4
2
2 3

1
STEP 2: Load vector
6 2t/m 6
Fixed-end forces {F}
0 0

6 6

STEP 3: Stiffness matrix


Stiffness relations [K]

STEP 4: Equilibrium {F} = [K] {D}


STEP 5: Equations solution Get {D}
STEP 6: Internal forces Q, N, and M

3-2- STEP 1: MODELING


a- Numbering

Organization of the solution requires numbering the members, joints, and


forces and displacements components. Several numbering schemes are
possible. Choice of the most appropriate scheme depends on several
considerations including the method of assembling and solving equations
of equilibrium (banded matrix methods, frontal method, etc.). When
banded matrix methods are used for solving equilibrium equations,
members may be numbered in any convenient order, but
joints should be numbered in such an order that
the maximum difference between the numbers of the two joints of each
member is as small as possible. For example, the numbering in Fig. Is
more appropriate than that in Fig.

The unknown displacement components are usually numbered in a


certain sequence (for example in plane frame; x, y, and rotation) starting
at joint 1 and proceeding in ascending order through the joints. These
displacement components are called degrees of freedom.
b- Degrees of Freedoms

Several models of skeletal structures may be used depending on the


nature of the structure and the loads. Common skeletal models include
space frames, space trusses, plane frames, plane trusses, beams, and grids.

* In space frames, each joint has six degrees of freedom: three


translations (in X, Y, and Z directions) and three rotations (about X, Y,
and Z axes).

* In space trusses, all member ends are assumed to have hinged


connections, i.e. three degrees of freedom at each joint (rotation about X,
Y, and Z axes).

* In plane frames, a joint has three degrees of freedom since it has two
translations and one rotation.

* In plane trusses, a joint has two degrees of freedom (the translation


only) since rotation are not considered.

* In beams, axial and transverse forces and displacements are uncoupled,


and separate analyses may be carried out for axial effects and transverse
effects.

* In grids, in-plane displacements are not considered; therefore, also


rotations about an axis normal to the plane of the grid are not considered.
Thus, a grid joint has only three degrees of freedom (an out-of-plane
translation, and two rotations about in-plane axes).
20 19 18 17 16 15 14 13 12 11

1 2 3 4 5 6 7 8 9 10

6 7 18 19
5
6 4 5 8 17 20
7 4 9 16 21
1 3 3 10 15 22
2 2 11 14 23
1 12 13 24

2 3 6 7 10 11 14 15 18 19

1 4 5 8 9 12 13 16 17 20

21 22 23 24
5
3 6 17 18 19 20
2 13 14 15 16
1 7 9 10 11 12
4 5 6 7 8
1 2 3 4

c- Local and Global Axes

The axes which are convenient in dealing with members individually are
called local (member) axes, but the axes which are convenient in dealing
with the structure as a whole are called global (structure) axes.
Displacement and force components may be expressed using one of the
previous two systems.

The relation between the components in the two systems of axes is


expressed in matrix form which called transformation matrix [T].
Y Y End 2
( X2,Y2,Z2)
End 2
( X2,Y2) End 1
Z
( X1,Y1,Z1)
Length =L
X

End 1 Y
( X1,Y1)

X X

Plane or grid member Space member


Z
a) Geometry
g g l l
f5 d5 f5 d5
g g l l
f4 d4 f4 d4
g g l l
g g f6 d6 f6 d6
f2 d2 l l
f2 d2
g g Global Local
f1 d1
g g l l l l
f3 d3 f1 d1 f3 d3

b) Plane frame member

g g l l
f4 d4 f4 d4
g g l l
f3 d3 f3 d3

g g
f2 d2 l l
f2 d2
g g Global Local
f1 d1
l l
f1 d1

c) Plane truss member

Global and local force and displacement components


• In the case of plane frame, if the components of member
end forces in global direction are:

{ f g } = { f 1g f 2g f 3g f 4g f 5g f 6g }

and the corresponding components in local directions are:

{ f l } = { f 1l f 2l f 3l f 4l f 5l f 6l }

as shown in Fig. , then;

{ f g } = [T ] { f l }

where;
c −s 0 0 0 0
s c 0 0 0 0
0 0 1 0 0 0
[T ] =
0 0 0 c −s 0
0 0 0 s c 0
0 0 0 0 0 1

in which; c = cos θ = (X2 – X1) / L , and


s = sin θ = (Y2 – Y1) / L
where (X1 , Y1) and (X2 , Y2) are the coordinates of the joints at the start
and the end of member respectively with respect to global axes, and L is
the length of the member.

Similarly, the relation between the displacement components in global


directions {dg} and the displacement components in local directions {dl}
for the joints of plane frame members is:

{d g } = [T ] { d l }
where [T] as defined above.

• In the case of plane trusses, each of {fg}, {fl}, {dg}, and


{dl} has only four elements and the transformation matrix
reduces to:
c −s 0 0
s c 0 0
[T ] =
0 0 c −s
0 0 s c

• In the case of grids, the transformation matrix is:

1 0 0 0 0 0
0 s c 0 0 0
0 −c s 0 0 0
[T ] =
0 0 0 1 0 0
0 0 0 0 s c
0 0 0 0 −c s

• In the case of space frames, each joint has six degrees


of freedom then the transformation matrix is:

t* φ φ φ
φ t *
φ φ
[T ] =
φ φ t* φ
φ φ φ t*

where

[Ø] = [0]3*3 , and

lx ly lz
[t ] = m x
*
my mz
nx ny nz

where (lx, mx, nx), (ly, my, ny), and (lz, mz, nz) are the direction cosines of
the local axes x, y, and z with respect to the global axes X, Y, and Z.

• In the case of space trusses, all member ends are


hinged. Therefore the transformation matrix is:
lx * * 0 0 0
mx * * 0 0 0
nx * * 0 0 0
[T ] =
0 0 0 lx * *
0 0 0 mx * *
0 0 0 nx * *

where the asterisks refer to unneeded elements, and (lx, mx, nx) are the
direction cosines which given by:

lx = (X2 – X1) / L, mx = (Y2 – Y1) / L, and nx = (Z2 – Z1) / L

3-2- STEP 2: LOAD VECTOR (MEMBER FIXED-END FORCES)


Member fixed-end forces means the reactions at the ends of members due
to loads, environmental changes, or support movement, when all the
unknown displacements at member end joints are prevented. These
reactions can be determined by classical methods such as column analogy
or consistent deformations.

Components of fixed-end forces in local directions may be arranged in a


vector { f ml } , which is called member load vector in local directions. The
corresponding components in global directions may be arranged in a
vector { f mg } which is called member load vector in global directions, and
can be determined from the transformation relation

{ f mg } = [T] { f ml }

Fig. Shows some cases of member fixed-end forces.

2 2
WL /12 WL /12 PL/8 PL/8
Wt/m P

L L
WL/2 WL/2 P/2 P/2
Wt/m Wt/m
2 2 2 2
WL /30 WL /20 5WL /96 5WL /96

L
0.167 WL 0.333 WL L

WL/4 WL/4
2 2 2 2
WL WL WL WL
( ) ( )
20 30 20 30
L L

2 2 2 2
2PL/9 2PL/9 P.a.b /L P.b.a /L
P P P

L a b
Pb/L Pa/L
P P

2 2 2 2
P.a.b P.b.a P.a.b P.b.a
( ) ( )
L2 L2 L2 L2
L L

M/4 M M/4

L
1.5 M / L 1.5 M / L
(1) (2)
P1
W

P1

P2 P2

(3) (4 )
P1

W
P3

P2
W

(5 )
3-3- STEP 3: STIFFNESS MATRIX (MEMBER END FORCES DUE TO
JOINT DISPLACEMENTS)
Displacements of joints cause member end displacements and hence
member deformations and internal forces between joints and member
ends. These member end forces depend on the type of connection
between joints and member end.

a- Force-Displacement Relation in Local Directions

• In the case of plane truss member shown in Fig. , let


the joints undergo displacements whose components in
local directions are d1l , d 2l , d 3l , and d 4l . The lateral
displacements can occur without deformation (small
displacements and hinged connections). On the other hand,
the axial displacements d1l and d 3l result in an elongation
( d 3l - d1l ) and hence tension N = ( d 3l - d1l ) EA / L.
The corresponding forces from joints to member ends are:
f dl1 = -N = ( d 3l - d 1l ) EA / L
f dl 2 = f dl 4 = 0
f dl3 = N = ( d 3l - d 1l ) EA / L
or in matrix form;
{ f dl } = [ k l ] { d l }
where
1 0 −1 0
0 0 0 0
[ kl ] =
−1 0 1 0
0 0 0 0
The matrix [ k l ] is known as the member stiffness matrix in local
directions. It can be noticed that the elements of the ith column of [ k l ]
are the forces { f dl } when d il = 1 while all other components of { d l } are
zeros. This observation is usually used as a convenient basis for deriving
the matrix [ k l ] for members of different types.

Consider a plane frame member with fixed ends as shown in Fig. . To


derive elements of the first column of [ k l ] , let d1l = 1 while all other
components of { d l } are zeros. The corresponding end forces are the
elements of the first column of [ k l ] , and these elements are:
(EA/L) { 1 0 0 -1 0 0 }T
Elements of the second column of [ k l ] are the forces correspond to d 2l =
1, while all other components of { d l } are zeros. Then the elements of the
second column of [ k l ] are:
(6EI/L2) { 0 2/L 1 0 -2/L 1 }T
Note that these forces can be determined using the method of consistent
deformations. Similarly the components of the third, fourth, fifth, and
sixth columns can be determined and the complete stiffness matrix of the
member in local axes directions is:
EA / L 0 0 − EA / L 0 0
0 12 EI / L3
6 EI / L2
0 − 12 EI / L
3
6 EI / L2
0 6 EI / L2 4 EI / L 0 − 6 EI / L2 2 EI / L
[k ]=
l

− EA / L 0 0 EA / L 0 0
0 − 12 EI / L3 − 6 EI / L2 0 12 EI / L3 − 6 EI / L2
0 6 EI / L2 2 EI / L 0 − 6 EI / L2 4 EI / L
It may be observed that each column of [ k l ] represent a set of forces in
equilibrium. It may also be observed that the matrix [ k l ] is symmetric,
i.e. k ijl = k lji . These two observations may be used for deriving some
elements of [ k l ] .
P1 D
3
D
W1 D 4
P2 2 2
Fixed 2 3

1
W2
D
1 1
Hinged

Structure and loads Members, joints and degrees


of freedom (D.O.F.)
global position
1 0 0 2 3 4 numbers 2 3 4 0 0 0
1 2 3 4 5 6 1 2 3 4 5 6 local column numbers

1 1 2 1
0 2 3 2
0 3 4 3

2 4 0 4
3 5 0 5
4 6 0 6

1 2
g g
K local row numbers K

Member stiffness matrices

1 1 2 1
0 2 3 2
0 3 4 3

2 4 0 4
3 5 0 5
4 6 0 6
1 2
g g
fm fm

Member load vectors


1 2 3 4
g1 g1 g1 g1 g1
1 k11 k14 k15 k16 D1 0 - fm1
2 g1
k41
g1 g2 g1
k44+k11 k45+k12 k46+k13
g2 g1 g2
D2 3 -
g1
fm4 - fm1
g2

=
3 g1 g2
g1 g1 g2 g1 g2 g1 g2
k51 k54+k21 k55+k22 k56+k23 D3 -5 - fm5 - f m2
4 g1
k61
g1 g2 g1
k64+k31 k65+k32 k66+k33
g2 g1 g2
D4 0 -
g1
fm6 - fm3
g2

Equilibrium equations

Example for assembly of equilibrium equations


To find the overall stiffness matrix
=1 2 2 4EI/L 2EI/L
6EI/L 6EI/L

=1 =1
EA/L EA/L

3 3 2 2
12EI/L 12EI/L 6EI/L 6EI/L

⎡ EA / L ⎤ ⎡ 0 ⎤ ⎡ 0 ⎤
⎢ 0 ⎥ ⎢ 3 ⎥ ⎢ 2 ⎥
⎢ ⎥ ⎢ 12 EI / L ⎥ ⎢ 6 EI / L ⎥
⎢ 0 ⎥ ⎢ 6 EI / L2 ⎥ ⎢ 4 EI / L ⎥
⎢ ⎥ ⎢ ⎥ ⎢ ⎥
⎢− EA / L ⎥ ⎢ 0 ⎥ ⎢ 0 ⎥
⎢ 0 ⎥ ⎢− 12 EI / L3 ⎥ ⎢− 6 EI / L2 ⎥
⎢ ⎥ ⎢ ⎥ ⎢ ⎥
⎢⎣ 0 ⎥⎦ 2
⎣⎢ 6 EI / L ⎦⎥ ⎣⎢ 2 EI / L ⎦⎥

= 1 6EI/L
2
6EI/L
2 2EI/L 4EI/L
=1
=1
EA/L EA/L

3 3 2 2
12EI/L 12EI/L 6EI/L 6EI/L

⎡− EA / L ⎤ ⎡ 0 ⎤ ⎡ 0 ⎤
⎢ 0 ⎥ ⎢ 3⎥ ⎢ 2 ⎥
⎢ ⎥ ⎢− 12 EI / L ⎥ ⎢ 6 EI / L ⎥
⎢ 0 ⎥ ⎢ − 6 EI / L2 ⎥ ⎢ 2 EI / L ⎥
⎢ ⎥ ⎢ ⎥ ⎢ ⎥
⎢ EA / L ⎥ ⎢ 0 ⎥ ⎢ 0 ⎥
⎢ 0 ⎥ ⎢ 12 EI / L3 ⎥ ⎢− 6 EI / L2 ⎥
⎢ ⎥ ⎢ ⎥ ⎢ ⎥
⎣⎢ 0 ⎦⎥ ⎢⎣ − 6 EI / L ⎥⎦
2
⎢⎣ 4 EI / L ⎥⎦

EA / L 0 0 − EA / L 0 0
0 12 EI / L3 6 EI / L2 0 − 12 EI / L3 6 EI / L2
0 6 EI / L2 4 EI / L 0 − 6 EI / L2 2 EI / L
[k ]=l

− EA / L 0 0 EA / L 0 0
0 − 12 EI / L3 − 6 EI / L2 0 12 EI / L3 − 6 EI / L2
0 6 EI / L2 2 EI / L 0 − 6 EI / L2 4 EI / L
=1 2
3EI/L
3EI/L
=1
EA/L EA/L =1

2 2
3 3 3EI/L 3EI/L
3EI/L 3EI/L

⎡ EA / L ⎤ ⎡ 0 ⎤ ⎡ 0 ⎤
⎢ 0 ⎥ ⎢ 3 ⎥ ⎢ 2 ⎥
⎢ ⎥ ⎢ 3EI / L ⎥ ⎢ 3EI / L ⎥
⎢ 0 ⎥ ⎢ 3EI / L2 ⎥ ⎢ 3EI / L ⎥
⎢ ⎥ ⎢ ⎥ ⎢ ⎥
⎢− EA / L ⎥ ⎢ 0 ⎥ ⎢ 0 ⎥
⎢ 0 ⎥ ⎢− 3EI / L3 ⎥ ⎢− 3EI / L2 ⎥
⎢ ⎥ ⎢ ⎥ ⎢ ⎥
⎢⎣ 0 ⎥⎦ ⎣⎢ 0 ⎦⎥ ⎣⎢ 0 ⎦⎥

2
=1 3EI/L

=1
EA/L EA/L =1

3 3
3EI/L 3EI/L

⎡− EA / L ⎤ ⎡ 0 ⎤ ⎡0 ⎤
⎢ 0 ⎥ ⎢ 3⎥ ⎢0 ⎥
⎢ ⎥ ⎢ − 3EI / L ⎥ ⎢ ⎥
⎢ 0 ⎥ ⎢− 3EI / L2 ⎥ ⎢0 ⎥
⎢ ⎥ ⎢ ⎥ ⎢ ⎥
⎢ EA / L ⎥ ⎢ 0 ⎥ ⎢0 ⎥
⎢ 0 ⎥ ⎢ 3EI / L3 ⎥ ⎢0 ⎥
⎢ ⎥ ⎢ ⎥ ⎢ ⎥
⎢⎣ 0 ⎥⎦ ⎣⎢ 0 ⎦⎥ ⎢⎣0⎥⎦

EA / L 0 0 − EA / L 0 0
0 3EI / L 3
3EI / L 2
0 − 3EI / L 3
0
0 3EI / L 2
3EI / L 0 − 3EI / L 2
0
[ kl ] =
− EA / L 0 0 EA / L 0 0
0 − 3EI / L3 − 3EI / L2 0 3EI / L3 0
0 0 0 0 0 0
2
=1 3EI/L 3EI/L

=1 =1

3 3 2 2
3EI/L 3EI/L 3EI/L 3EI/L

⎡0 ⎤ ⎡ 0 ⎤ ⎡ 0 ⎤
⎢0 ⎥ ⎢ 3 ⎥ ⎢ 2 ⎥
⎢ ⎥ ⎢ 3EI / L ⎥ ⎢ 3EI / L ⎥
⎢0 ⎥ ⎢ 3EI / L2 ⎥ ⎢ 3EI / L ⎥
⎢ ⎥ ⎢ ⎥ ⎢ ⎥
⎢0 ⎥ ⎢ 0 ⎥ ⎢ 0 ⎥
⎢0 ⎥ ⎢− 3EI / L3 ⎥ ⎢− 3EI / L2 ⎥
⎢ ⎥ ⎢ ⎥ ⎢ ⎥
⎣⎢0⎦⎥ ⎢⎣ 0 ⎥⎦ ⎢⎣ 0 ⎥⎦

2
=1 3EI/L
=1
EA/L EA/L =1

3 3
3EI/L 3EI/L

⎡− EA / L ⎤ ⎡ 0 ⎤ ⎡0 ⎤
⎢ 0 ⎥ ⎢ 3⎥ ⎢0 ⎥
⎢ ⎥ ⎢ − 3EI / L ⎥ ⎢ ⎥
⎢ 0 ⎥ ⎢− 3EI / L2 ⎥ ⎢0 ⎥
⎢ ⎥ ⎢ ⎥ ⎢ ⎥
⎢ EA / L ⎥ ⎢ 0 ⎥ ⎢0 ⎥
⎢ 0 ⎥ ⎢ 3EI / L3 ⎥ ⎢0 ⎥
⎢ ⎥ ⎢ ⎥ ⎢ ⎥
⎢⎣ 0 ⎥⎦ ⎣⎢ 0 ⎦⎥ ⎢⎣0⎥⎦

0 0 0 − EA / L 0 0
0 3EI / L 3
3EI / L 2
0 − 3EI / L 3
0
0 3EI / L 2
3EI / L 0 − 3EI / L 2
0
[ kl ] =
0 0 0 EA / L 0 0
0 − 3EI / L3 − 3EI / L2 0 3EI / L3 0
0 0 0 0 0 0
=1

EA/L EA/L =1 =1

⎡ EA / L ⎤ ⎡0 ⎤ ⎡0 ⎤
⎢ 0 ⎥ ⎢0 ⎥ ⎢0 ⎥
⎢ ⎥ ⎢ ⎥ ⎢ ⎥
⎢ 0 ⎥ ⎢0 ⎥ ⎢0 ⎥
⎢ ⎥ ⎢ ⎥ ⎢ ⎥
⎢− EA / L ⎥ ⎢0 ⎥ ⎢0 ⎥
⎢ 0 ⎥ ⎢0 ⎥ ⎢0 ⎥
⎢ ⎥ ⎢ ⎥ ⎢ ⎥
⎢⎣ 0 ⎥⎦ ⎢⎣0⎥⎦ ⎢⎣0⎥⎦

=1

EA/L EA/L =1 =1

⎡− EA / L ⎤ ⎡0 ⎤ ⎡0 ⎤
⎢ 0 ⎥ ⎢0 ⎥ ⎢0 ⎥
⎢ ⎥ ⎢ ⎥ ⎢ ⎥
⎢ 0 ⎥ ⎢0 ⎥ ⎢0 ⎥
⎢ ⎥ ⎢ ⎥ ⎢ ⎥
⎢ EA / L ⎥ ⎢0 ⎥ ⎢0 ⎥
⎢ 0 ⎥ ⎢0 ⎥ ⎢0 ⎥
⎢ ⎥ ⎢ ⎥ ⎢ ⎥
⎢⎣ 0 ⎥⎦ ⎢⎣0⎥⎦ ⎢⎣0⎥⎦

EA / L 0 0 − EA / L 0 0
0 0 0 0 0 0
0 0 0 0 0 0
[ kl ] =
− EA / L 0 0 EA / L 0 0
0 0 0 0 0 0
0 0 0 0 0 0
3-4- STEP 4 : The Overall Equilibrium Equation

From the steps number 3 & 4 we can construct the overall equilibrium
equation as follow:

{ F } nx1 = [ K ] nxn * { D }nx1


Where: (n) the number of degree of freedom (DOF) which defined
from step no 1( modeling ).

3-5- STEP 5 : Solve The Equilibrium Equation

By using back Guss elimination we can solve the equilibrium


equation and find the overall global displacement { D }g
We can find the local displacement from the relation

{ D }l = [ T ]T * { D }g

Where [ T ] the transformation matrix for member

For plane truss member is:

c −s 0 0
s c 0 0
[T ] =
0 0 c −s
0 0 s c

For plane frame member is:

c −s 0 0 0 0
s c 0 0 0 0
0 0 1 0 0 0
[T ] =
0 0 0 c −s 0
0 0 0 s c 0
0 0 0 0 0 1
3-6- STEP 6 : Find The Internal Forces

For plane truss member:

EA
N= Δ Δ = (d 3 − d1 ) * cosθ + (d 4 − d 2 ) * sin θ
L
d4
d3

d2

For plane frame member:


d1

Dg1 = D1 * cosθ + D2 * sin θ

Dg 2 = D2 * cosθ − D1 * sin θ

Dg 3 = D3

Dg 4 = D4 * cos θ + D5 * sin θ

Dg 5 = D5 * cos θ − D4 * sin θ

Dg 6 = D6

EA
N1 = N 2 = ( Dg 4 − Dg1 )
L

12 EI 12 EI
S1 = S 2 = ( Dg 2 − Dg 5 ) + ( Dg 6 + Dg3 )
L3 L2

6 EI 2 EI
M1 = ( Dg 5 − Dg 2 ) − ( Dg 6 + 2 Dg3 )
L2 L
6 EI 2 EI
M2 = ( Dg 2 − Dg 5 ) + ( Dg 3 + 2 Dg 6 )
L2 L
CHAPTER (6)

Analysis of Plane Trusses


By Using Stiffness Matrix Method

6-1- STEP 1: MODELING

a- Numbering

Organization of the solution requires numbering the members, joints,


and forces and displacements components. Several numbering schemes are
possible. Choice of the most appropriate scheme depends on several
considerations including the method of assembling and solving equations of
equilibrium (banded matrix methods, frontal method, etc.). When banded
matrix methods are used for solving equilibrium equations, members may be
numbered in any convenient order, but joints should be numbered in such an
order that the maximum difference between the numbers of the two joints of
each member is as small as possible. For example, the numbering in Figure
(b) is more appropriate than that in Figure (a)

20 19 18 17 16 15 14 13 12 11

1 2 3 4 5 6 7 8 9 10
(a)

2 3 6 7 10 11 14 15 18 19

1 4 5 8 9 12 13 16 17 20

(b)
6 7 8

1 5
2 3 4
(a)

2 5 6

1 8
3 4 7
(b)
The unknown displacement components are usually numbered in a
certain sequence starting at joint 1 and proceeding in ascending order
through the joints. These displacement components are called degrees of
freedom.

b- Degrees of Freedoms

Several models of skeletal structures may be used depending on the


nature of the structure and the loads. Common skeletal models include
space frames, space trusses, plane frames, plane trusses, beams, and grids.

* In plane trusses, a joint has two degrees of freedom (the translation


only) since rotation are not considered.

* In beams, axial and transverse forces and displacements are uncoupled,


and separate analyses may be carried out for axial effects and transverse
effects.
c- Local and Global Axes

The axes which are convenient in dealing with members individually are
called local (member) axes, but the axes which are convenient in dealing
with the structure as a whole are called global (structure) axes.
Displacement and force components may be expressed using one of the
previous two systems.

The relation between the components in the two systems of axes is


expressed in matrix form which called transformation matrix [T].

In the case of plane trusses, each of {fg}, {fl}, {dg}, and {dl} has only
four elements and the transformation matrix reduces to: dg4

dl4 dl3
c −s 0 0 dg3
s c 0 0
[T ] =
0 0 c −s
0 0 s c dg2

dl2 dl1
dg1

in which;
c = cos θ = (X2 – X1) / L
, and
s = sin θ = (Y2 – Y1) / L

where (X1 , Y1) and (X2 , Y2) are the coordinates of the joints at the start
and the end of member respectively with respect to global axes, and L is
the length of the member.

⎧ d1 ⎫ c −s 0 0
⎪d ⎪
⎪ ⎪ s c 0 0
{d }l = ⎨ 2⎬ [ T] =
⎪d 3 ⎪ 0 0 c −s
⎪⎩d 4 ⎪⎭ 0 0 s c
Similarly, the relation between the displacement components in global
directions {dg} and the displacement components in local directions {dl}
for the joints of plane frame members is:

{d g } = [T ] { d l }

where [T] as defined above.

6-2- STEP 2: LOAD VECTOR (MEMBER FIXED-END FORCES)

A member fixed-end force means the reactions at the ends of


members due to loads, environmental changes, or support movement,
when all the unknown displacements at member end joints are prevented.
These reactions can be determined by classical methods such as
consistent deformations.

Components of fixed-end forces in local directions may be arranged in a


vector { f ml } , which is called member load vector in local directions. The
corresponding components in global directions may be arranged in a
vector { f mg } which is called member load vector in global directions, and
can be determined from the transformation relation.

{ f mg } = [T] { f ml }

Example (1)
Construct the modeling and find
the over all load vector for the
given plane trusses.
10 t

4.00

8t
5t
4.00
D3
Solution 10 t
D2 4
4
2
Q D.O.F. = 4

6
∴ {F }4 x1 = [K ]4 x 4 {D}4 x1 4.00

2
5
D4

⎡ 8 ⎤
8t 1 3 3
⎢ 0 ⎥ D1 5t
∴ {F }4 x1 =⎢ ⎥ 4.00
⎢− 10⎥
⎢ ⎥
⎣ −5⎦

Example (2)

Construct the modeling and find the over all load vector for the given
8t 8t 8t

6t

3.00

6t
4.00 4.00
plane trusses.
Solution

8t 8t 8t
2 3 3 4 5
6t
10 8 9
3.00
5

1 4 6
1 2
6t
4.00 4.00
D. O.F. = 4
∴ {F }4 x1 = [K ]4 x 4 {D}4 x1

Overall Load Vector: { F }4x1 =

6-3- STEP 3: STIFFNESS MATRIX (MEMBER END FORCES


DUE TO JOINT DISPLACEMENTS)

Displacements of joints cause member end displacements and hence


member deformations and internal forces between joints and member
ends. These member end forces depend on the type of connection
between joints and member end.

a- Force-Displacement Relation in Local Directions

• In the case of plane truss member shown in Fig., let the joints
undergo displacements whose components in local directions are
d1l , d 2l , d 3l , and d 4l . The lateral displacements can occur without
deformation (small displacements and hinged connections). On the
other hand, the axial displacements d1l and d 3l result in an
elongation ( d 3l - d1l ) and hence tension

N = ( d 3l - d1l ) EA / L.

The corresponding forces from joints to member ends are:

f dl1 = -N = ( d 3l - d1l ) EA / L

f dl 2 = f dl 4 = 0

f dl3 = N = ( d 3l - d1l ) EA / L
or in matrix form;

{ f dl } = [ k l
] { d l
}

⎡ EA/ L ⎤ ⎡0⎤ ⎡− EA/ L⎤ ⎡0⎤


⎢ 0 ⎥ ⎢ ⎥⎢ 0 ⎥ ⎢ ⎥
Where: [ kl ] =
⎢ ⎥ ⎢0⎥ ⎢ ⎥ ⎢0⎥
⎢− EA/ L⎥ ⎢0⎥ ⎢ EA/ L ⎥ ⎢0⎥
⎢ ⎥ ⎢ ⎥⎢ ⎥⎢ ⎥
⎣ 0 ⎦ ⎣0⎦ ⎣ 0 ⎦ ⎣0⎦

=1
EA/L EA/L =1

⎡ EA/ L ⎤ ⎡0⎤
⎢ 0 ⎥ ⎢0⎥
⎢ ⎥ ⎢ ⎥
⎢− EA/ L⎥ ⎢0⎥
⎢ ⎥ ⎢ ⎥
⎣ 0 ⎦ ⎣0⎦
=1
EA/L EA/L
=1

⎡− EA/ L⎤ ⎡0⎤
⎢ 0 ⎥ ⎢0⎥
⎢ ⎥ ⎢ ⎥
⎢ EA/ L ⎥ ⎢0⎥
⎢ ⎥ ⎢ ⎥
⎣ 0 ⎦ ⎣0⎦

Then; The local stiffness Matrix for any plane truss member equal:
⎡1 0 − 1 0⎤
⎢0 0 0 0⎥⎥
[ ]
kl =
EA ⎢
L ⎢− 1 0 1 0⎥
⎢ ⎥
⎣0 0 0 0⎦

The matrix [ k l ] is known as the member stiffness matrix in local


directions. It can be noticed that the elements of the ith column of [ k l ]
are the forces { f dl } when d il = 1 while all other components of { d l } are
zeros. This observation is usually used as a convenient basis for deriving
the matrix [ k l ] for members of different types.

It may be observed that each column of [ k l ] represent a set of forces in


equilibrium. It may also be observed that the matrix [ k l ] is symmetric,
i.e. k ijl = k lji . These two observations may be used for deriving some
elements of [ k l ] .

6-4- STEP 4: The Overall Equilibrium Equation

From the step no 2 & 3 we can construct the equation as follow:


{ F }nx1 = [ K ] n x n { D } nx1 ……..Where n is the DOF
by solution the overall equilibrium equation we get the values of {D}.

6-5- STEP 6: The internal forces for plane truss member

By using the known global displacement we can determined the internal


forces for plane trusses as follows:

EA
N= Δ where: Δ = (d 3 − d1 ) * cos θ + (d 4 − d 2 ) * sin θ
L
Solved Examples
10 t
Example3:

The following truss has


constant EA, determine
the reactions and 4.00
member forces.
8t
5t
4.00

D3
Solution 10 t
D2 4
4
2
Q D.O.F. = 4

6
∴ {F }4 x1 = [K ]4 x 4 {D}4 x1 1
4.00

2
5
D4

⎡ 8 ⎤
8t 1 3 3
⎢ 0 ⎥ D1 5t
∴ {F }4 x1 =⎢ ⎥ 4.00
⎢− 10⎥
⎢ ⎥
⎣ −5⎦

For element 1: L = 4.0 m, θ = 90˚ For element 2: L = 4.0 m, θ = 90˚


⎡0 0 0 0⎤ ⎡0 0 0 0⎤
⎢ − 1⎥⎥ ⎢ − 1⎥⎥
EA ⎢ 0 1 0 EA ⎢ 0 1 0
[k1 ]g = [k 2 ]g =
4 ⎢0 0 0 0⎥ 4 ⎢0 0 0 0⎥
⎢ ⎥ ⎢ ⎥
⎣0 −1 0 1⎦ ⎣0 −1 0 1⎦

For element 3: L = 4.0 m, θ = 0.0˚ For element 4: L = 4.0 m, θ = 0.0˚


⎡1 0 −1 0 ⎤ ⎡1 0 −1 0 ⎤
⎢ ⎥ ⎢ ⎥
EA ⎢ 0 0 0 0 EA ⎢ 0 0 0 0
[k 3 ]g = [k 3 ]l = ⎥ [k 4 ]g = [k 4 ]l = ⎥
4 ⎢− 1 0 1 0 ⎥ 4 ⎢− 1 0 1 0 ⎥
⎢ ⎥ ⎢ ⎥
⎣0 0 0 0 ⎦ ⎣0 0 0 0 ⎦
For element 5: L = 4 / 2 m, θ = 45˚ For element 6: L = 4 / 2 m, θ = 315˚
⎡0.5 − − − ⎤ ⎡ 0.5 − 0.5 − 0.5 0.5 ⎤
⎢ ⎥ ⎢ 0.5 − 0.5⎥⎥
EA ⎢ − − − − EA ⎢− 0.5 0.5
[k 5 ]g = ⎥ [k 6 ]g =
4/ 2 ⎢ − − − − ⎥ 4 / 2 ⎢− 0.5 0.5 0.5 − 0.5⎥
⎢ ⎥ ⎢ ⎥
⎣− − − − ⎦ ⎣ 0.5 − 0.5 − 0.5 0.5 ⎦

Overall equilibrium equation:

⎧ 8 ⎫ ⎡ 0.338 0 0 0 ⎤ ⎧ D1 ⎫
⎪ 0 ⎪ ⎢ 0
⎪ ⎪ ⎢ 0.338 0.088 ⎥⎥ ⎪⎪ D2 ⎪⎪
− 0.088
⎨ ⎬= ⎨ ⎬
⎪ − 10 ⎪ ⎢ 0 − 0.088 0.338 − 0.088⎥ ⎪ D3 ⎪
⎪⎩ − 5 ⎪⎭ ⎢⎣ 0 ⎥
0.088 − 0.088 0.338 ⎦ ⎪⎩ D4 ⎪⎭

By solving 4 equations; then

⎧ 23.66 ⎫
⎪ EA ⎪
⎧ D1 ⎫ ⎪ ⎪
⎪ ⎪ ⎪ ⎪
⎪ ⎪ ⎪ ⎪
⎪ − 3.42 ⎪
⎪D ⎪
⎪ EA ⎪
⎪⎪ ⎪⎪ ⎪
2

⎨ ⎬=⎨ ⎬
⎪ D3 ⎪ ⎪ − 36.57 ⎪
⎪ ⎪ ⎪ ⎪
⎪ ⎪ ⎪ EA ⎪
⎪ D4 ⎪ ⎪ ⎪
⎪⎩ ⎪⎭ ⎪ − 23.42 ⎪
⎪ ⎪
⎪⎩ EA ⎪⎭
For elements:
− 36.57
Δ 1 = ( − 3.42 − 23.66 ) × cos 90 + ( −0)
EA
− 36.57
then; Δ1 =
EA
23.42 − 23.66 3.42 − 16.73
similarly; Δ2 = , Δ3 = , Δ4 = , Δ5 = ,
EA EA EA EA
− 6.88
Δ6 =
EA
then;
EA
F=Δ
L

− 36.57 EA
F1 = * = −9.14 t
EA 4
23.42 EA
F2 = * = 5.85 t
EA 4
− 23 . 66 EA
F3 = * = − 5 . 91 t
EA 4
3.42 EA
F4 = * = 0.855 t
EA 4

− 16.73 EA
F5 = * = −2.95 t
EA 4 2

− 6.88 EA
F6 = * = −1.21 t
EA 4 2

-5.91 t
-1

-9.14 t 5.85 t
t
5

.21
.9
-2

0.855 t

The Final internal forces


CHAPTER (7)

Analysis of Plane Frames


By Using Stiffness Matrix Method

7-1- STEP 1: MODELING


a- Numbering

As we indicated in the previous chapter, organization of the solution


requires numbering the members, joints, and forces and displacements
components. Several numbering schemes are possible. Choice of the most
appropriate scheme depends on several considerations including the method
of assembling and solving equations of equilibrium (banded matrix methods,
frontal method, etc.). When banded matrix methods are used for solving
equilibrium equations, members may be numbered in any convenient order,
but joints should be numbered in such an order that the maximum difference
between the numbers of the two joints of each member is as small as
possible. For example, the numbering in Figure (b) is more appropriate than
that in Figure (a)

1 6 11 13 14 15
19 20 19 20

10 11 12 10 11 12
7 11
2 12 10 12
17 18 17 18

7 8 9 7 8 9
8 8
3 13 7 9
15 16 15 16

4 5 6 4 5 6

9 5
4 14 4 6
13 14 13 14

1 2 3 1 2 3

5 10 15 1 2 3

(a) (b)
The unknown displacement components are usually numbered in a certain
sequence (for example in plane frame; x, y, and rotation) starting at joint 1
and proceeding in ascending order through the joints. These displacement
components are called degrees of freedom.

b- Degrees of Freedoms

Several models of skeletal structures may be used depending on the


nature of the structure and the loads. Common skeletal models include
space frames, space trusses, plane frames, plane trusses, beams, and grids.

* In plane frames, a free node has three degrees of freedom since it has
two translations and one rotation ( Δx , Δy & θ ).

* In beams, axial and transverse forces and displacements are uncoupled,


and separate analyses may be carried out for axial effects and transverse
effects.

c- Local and Global Axes

The axes which are convenient in dealing with members individually are
called local (member) axes, but the axes which are convenient in dealing
with the structure as a whole are called global (structure) axes.
Displacement and force components may be expressed using one of the
previous two systems.

dg4

dl4 dl3
dg3

dg2

dl2 dl1
dg1

The relation between the components in the two systems of axes is


expressed in matrix form which called transformation matrix [T].
• In the case of plane frame, if the components of member end
forces in global direction are:

{ f g } = { f1g f 2g f 3g f 4g f 5g f 6g }

and the corresponding components in local directions are:

{ f l } = { f1l f 2l f 3l f 4l f 5l f 6l }

as shown in Fig. , then;

{ f g } = [T ] { f l }

where;

c −s 0 0 0 0
s c 0 0 0 0
0 0 1 0 0 0
[T ] =
0 0 0 c −s 0
0 0 0 s c 0
0 0 0 0 0 1

in which; c = cos θ = (X2 – X1) / L ,

and s = sin θ = (Y2 – Y1) / L

where (X1 , Y1) and (X2 , Y2) are the coordinates of the joints at the start
and the end of member respectively with respect to global axes, and L is
the length of the member.

Similarly, the relation between the displacement components in global


directions {dg} and the displacement components in local directions {dl}
for the joints of plane frame members is:

{d g } = [T ] { d l }
where [T] as defined above.
7-2- STEP 2: LOAD VECTOR (MEMBER FIXED-END FORCES)

A member fixed-end force means the reactions at the ends of members


due to loads, environmental changes, or support movement, when all the
unknown displacements at member end joints are prevented. These
reactions can be determined by classical methods such as column analogy
or consistent deformations.

Components of fixed-end forces in local directions may be arranged in a


vector { f ml } , which is called member load vector in local directions. The
corresponding components in global directions may be arranged in a
vector { f mg } which is called member load vector in global directions, and
can be determined from the transformation relation

{ f mg } = [T] { f ml }

Some cases of member fixed-end forces.

2 2
WL /12 WL /12 PL/8 PL/8
Wt/m P

L L
WL/2 WL/2 P/2 P/2

Wt/m Wt/m
2 2 2 2
WL /30 WL /20 5WL /96 5WL /96

L
0.167 WL 0.333 WL L

WL/4 WL/4
2 2 2 2
WL WL WL WL
( ) ( )
20 30 20 30
L L
No7
CASE OF LOCAL LOAD
LOADING VECTFOR
l
{f} MEMBER
= [0 1/6wL-wL / 60
m
2
0 1/6wL wL /60 2
]T
L

1
{f}lm = [0 wL/30 -wL2/60 0 wL/60 wL2/30 ]T
L {f}lm = [ 0 wL/2 -wL2/12 0 wL/2 wL2/12]T

9 L

{f}lm = [0 wL/60 -wL2/30 0 wL/30 wL2/60 ]T

2 L
{fm}l = [ 0 wb/2 - wbL/24 (3-b2/L2 )
a b a
0 wb/2 wbL/24 (3-b2/L2 ) ]T
10 {f}lm = [0 wL/15 -wL2/15 0 wL/20 wL2/20 ]T
3
L

11 a b a {f}lm = [ 0 wa -wa2/6 (3-2a /L )

{f}lm = [0 0wL/20 wa22/20


wa -wL /6 ×(3-2a T 2
/L ) ]wL
0 wL/15 /15 ]T

4 L
{f}lm=[ 0 1/4wL -5/96wL2 0 1/4wL 5/96w L2 ] T

12 L/2 L/2
{f}lm = [0 wc*a/a+b -wc/L2 (ab2 –c2/6(b-a/2))

a-c/2 c b-c/2
5 0 wc*b/a+b wc/L2 (ab2 –c2/6 (a-b/2))]T

{f}lm = [ 0 (b+L/4 )× w – w/12 (L2-a2 (2-a/L)


L/2 L/2
13 0 ( b+L/4 )×w w/12 (L2-a2 (2-a/L))]T

6 {f}lm = 0 [ wa* (a/2+b)/a+b -wa2/4[2-a/L(8/3-


a b
a/L)
{f}lm =0[0wa* (a/2)/(a+b)
1/3wL
3
-wL2/15wa0 /4 [2-a/L(8/3-
1/3wL ]T
wL2/15 ]T
L
14
{f}lm = [0 wb/15 -wL2/15 0 wL/20 wL2/20 ]T

a b

15
{f}lm = [0 1/3 wL -wL2/20 0 1/6 wL wL2/30]T

16
{f}lm = [0 1/6 wL -wL2/30 0 1/3 wL wL2/20 ]T

17

{f}lm = [0 1/6 wL -wL2/30 0 1/3 wL wL2/20 ]T


a b

18

{f}lm = [0 q*a/2(b+2q/3L) –q*a2/6(1-


a b
a/L+3/10*a2/L2) 0 q*a/2 (q-3a) q*a2/60
*a/L(5.3a/L) ]T

19

{f}lm = [0 q*a/2(b+a/3L) –q*a2(1/3-


2 2
a b
a/2L+a /5L 0 q*a/2(2q/3L) –q*a2(1/3-

20
P
{f}lm = [0 P/2 -PL/8 0 P/2 PL/8]T
L/2 L/2

21
P
{f}lm = [0 P b/L -Pa b 2/L2 0 P a/L
a b Pb a 2/L2 ]T
22
P P {f }lm =[0 P -P*a/L(L-a)

0 P -P*a/L(L-a) ]T
a L-2a a

23
P P P {f }lm =[0 3P/2 -5PL/16

a a a a
0 3P/2 5PL/16) ]T

24
P P P P {f }lm =[0 2P -5PL/16

a a a a a
0 2P 5PL/16) ]T

25
P P P P P P
{ f }lm =[0 6P/2 -P*L/24(2n+1/n)
a a a a a
a/2 a/2
0 6P/2 P*L/24(2n+1/n) ]T
7-3- STEP 3: STIFFNESS MATRIX (MEMBER END FORCES
DUE TO JOINT DISPLACEMENTS)

Displacements of joints cause member end displacements and hence


member deformations and internal forces between joints and member
ends. These member end forces depend on the type of connection
between joints and member end.

a- Force-Displacement Relation in Local Directions

Consider a plane frame member with fixed ends as shown in Fig. . To


derive elements of the first column of [ k l ] , let d1l = 1 while all other
components of { d l } are zeros. The corresponding end forces are the
elements of the first column of [ k l ] , and these elements are:

(EA/L) { 1 0 0 -1 0 0 }T

Elements of the second column of [ k l ] are the forces correspond to d 2l =


1, while all other components of { d l } are zeros. Then the elements of the
second column of [ k l ] are:

(6EI/L2) { 0 2/L 1 0 -2/L 1 }T

Note that these forces can be determined using the method of consistent
deformations. Similarly the components of the third, fourth, fifth, and
sixth columns can be determined and the complete stiffness matrix of the
member in local axes directions is:

EA / L 0 0 − EA / L 0 0
0 12 EI / L3 6 EI / L2 0 − 12 EI / L3 6 EI / L2
0 6 EI / L2 4 EI / L 0 − 6 EI / L2 2 EI / L
[ kl ] =
− EA / L 0 0 EA / L 0 0
0 − 12 EI / L 3
− 6 EI / L 2
0 12 EI / L3
− 6 EI / L2
0 6 EI / L2 2 EI / L 0 − 6 EI / L2 4 EI / L

It may be observed that each column of [ k l ] represent a set of forces in


equilibrium. It may also be observed that the matrix [ k l ] is symmetric,
i.e. k ijl = k lji . These two observations may be used for deriving some
elements of [ k l ] .
=1 2 2 4EI/L 2EI/L
6EI/L 6EI/L

=1 =1
EA/L EA/L

3 3 2 2
12EI/L 12EI/L 6EI/L 6EI/L

⎡ EA / L ⎤ ⎡ 0 ⎤ ⎡ 0 ⎤
⎢ 0 ⎥ ⎢ 3 ⎥ ⎢ 2 ⎥
⎢ ⎥ ⎢ 12 EI / L ⎥ ⎢ 6 EI / L ⎥
⎢ 0 ⎥ ⎢ 6 EI / L2 ⎥ ⎢ 4 EI / L ⎥
⎢ ⎥ ⎢ ⎥ ⎢ ⎥
⎢− EA / L ⎥ ⎢ 0 ⎥ ⎢ 0 ⎥
⎢ 0 ⎥ ⎢− 12 EI / L3 ⎥ ⎢− 6 EI / L2 ⎥
⎢ ⎥ ⎢ ⎥ ⎢ ⎥
⎢⎣ 0 ⎥⎦ ⎣⎢ 6 EI / L ⎦⎥
2
⎣⎢ 2 EI / L ⎦⎥

2
= 1 6EI/L 6EI/L
2 2EI/L 4EI/L
=1
=1
EA/L EA/L

3 3 2 2
12EI/L 12EI/L 6EI/L 6EI/L

⎡− EA / L ⎤ ⎡ 0 ⎤ ⎡ 0 ⎤
⎢ 0 ⎥ ⎢ 3⎥ ⎢ 2 ⎥
⎢ ⎥ ⎢− 12 EI / L ⎥ ⎢ 6 EI / L ⎥
⎢ 0 ⎥ ⎢ − 6 EI / L2 ⎥ ⎢ 2 EI / L ⎥
⎢ ⎥ ⎢ ⎥ ⎢ ⎥
⎢ EA / L ⎥ ⎢ 0 ⎥ ⎢ 0 ⎥
⎢ 0 ⎥ ⎢ 12 EI / L3 ⎥ ⎢− 6 EI / L2 ⎥
⎢ ⎥ ⎢ ⎥ ⎢ ⎥
⎣⎢ 0 ⎦⎥ ⎢⎣ − 6 EI / L ⎥⎦ ⎢⎣ 4 EI / L ⎥⎦
2

EA / L 0 0 − EA / L 0 0
0 12 EI / L3 6 EI / L2 0 − 12 EI / L3 6 EI / L2
0 6 EI / L2 4 EI / L 0 − 6 EI / L2 2 EI / L
[ kl ] =
− EA / L 0 0 EA / L 0 0
0 − 12 EI / L3 − 6 EI / L2 0 12 EI / L3 − 6 EI / L2
0 6 EI / L2 2 EI / L 0 − 6 EI / L2 4 EI / L
=1 2
3EI/L
3EI/L
=1
EA/L EA/L =1

2 2
3 3 3EI/L 3EI/L
3EI/L 3EI/L

⎡ EA / L ⎤ ⎡ 0 ⎤ ⎡ 0 ⎤
⎢ 0 ⎥ ⎢ 3 ⎥ ⎢ 2 ⎥
⎢ ⎥ ⎢ 3EI / L ⎥ ⎢ 3EI / L ⎥
⎢ 0 ⎥ ⎢ 3EI / L2 ⎥ ⎢ 3EI / L ⎥
⎢ ⎥ ⎢ ⎥ ⎢ ⎥
⎢− EA / L ⎥ ⎢ 0 ⎥ ⎢ 0 ⎥
⎢ 0 ⎥ ⎢− 3EI / L3 ⎥ ⎢− 3EI / L2 ⎥
⎢ ⎥ ⎢ ⎥ ⎢ ⎥
⎣⎢ 0 ⎦⎥ ⎢⎣ 0 ⎥⎦ ⎢⎣ 0 ⎥⎦

2
=1 3EI/L

=1
EA/L EA/L =1

3 3
3EI/L 3EI/L

⎡− EA / L ⎤ ⎡ 0 ⎤ ⎡0 ⎤
⎢ 0 ⎥ ⎢ 3⎥ ⎢0 ⎥
⎢ ⎥ ⎢ − 3EI / L ⎥ ⎢ ⎥
⎢ 0 ⎥ ⎢− 3EI / L2 ⎥ ⎢0 ⎥
⎢ ⎥ ⎢ ⎥ ⎢ ⎥
⎢ EA / L ⎥ ⎢ 0 ⎥ ⎢0 ⎥
⎢ 0 ⎥ ⎢ 3EI / L3 ⎥ ⎢0 ⎥
⎢ ⎥ ⎢ ⎥ ⎢ ⎥
⎣⎢ 0 ⎦⎥ ⎣⎢ 0 ⎦⎥ ⎣⎢0⎦⎥

EA / L 0 0 − EA / L 0 0
0 3EI / L3 3EI / L2 0 − 3EI / L3 0
0 3EI / L2 3EI / L 0 − 3EI / L2 0
[ kl ] =
− EA / L 0 0 EA / L 0 0
0 − 3EI / L3 − 3EI / L2 0 3EI / L3 0
0 0 0 0 0 0
2
=1 3EI/L 3EI/L

=1 =1

3 3 2 2
3EI/L 3EI/L 3EI/L 3EI/L

⎡0 ⎤ ⎡ 0 ⎤ ⎡ 0 ⎤
⎢0 ⎥ ⎢ 3 ⎥ ⎢ 2 ⎥
⎢ ⎥ ⎢ 3EI / L ⎥ ⎢ 3EI / L ⎥
⎢0 ⎥ ⎢ 3EI / L2 ⎥ ⎢ 3EI / L ⎥
⎢ ⎥ ⎢ ⎥ ⎢ ⎥
⎢0 ⎥ ⎢ 0 ⎥ ⎢ 0 ⎥
⎢0 ⎥ ⎢− 3EI / L3 ⎥ ⎢− 3EI / L2 ⎥
⎢ ⎥ ⎢ ⎥ ⎢ ⎥
⎣⎢0⎦⎥ ⎢⎣ 0 ⎥⎦ ⎢⎣ 0 ⎥⎦

2
=1 3EI/L
=1
EA/L EA/L =1

3 3
3EI/L 3EI/L

⎡− EA / L ⎤ ⎡ 0 ⎤ ⎡0 ⎤
⎢ 0 ⎥ ⎢ 3⎥ ⎢0 ⎥
⎢ ⎥ ⎢ − 3EI / L ⎥ ⎢ ⎥
⎢ 0 ⎥ ⎢− 3EI / L2 ⎥ ⎢0 ⎥
⎢ ⎥ ⎢ ⎥ ⎢ ⎥
⎢ EA / L ⎥ ⎢ 0 ⎥ ⎢0 ⎥
⎢ 0 ⎥ ⎢ 3EI / L3 ⎥ ⎢0 ⎥
⎢ ⎥ ⎢ ⎥ ⎢ ⎥
⎢⎣ 0 ⎥⎦ ⎣⎢ 0 ⎦⎥ ⎢⎣0⎥⎦

0 0 0 − EA / L 0 0
0 3EI / L3 3EI / L2 0 − 3EI / L3 0
0 3EI / L2 3EI / L 0 − 3EI / L2 0
[ kl ] =
0 0 0 EA / L 0 0
0 − 3EI / L3 − 3EI / L2 0 3EI / L3 0
0 0 0 0 0 0
=1

EA/L EA/L =1 =1

⎡ EA / L ⎤ ⎡0 ⎤ ⎡0 ⎤
⎢ 0 ⎥ ⎢0 ⎥ ⎢0 ⎥
⎢ ⎥ ⎢ ⎥ ⎢ ⎥
⎢ 0 ⎥ ⎢0 ⎥ ⎢0 ⎥
⎢ ⎥ ⎢ ⎥ ⎢ ⎥
⎢− EA / L ⎥ ⎢0 ⎥ ⎢0 ⎥
⎢ 0 ⎥ ⎢0 ⎥ ⎢0 ⎥
⎢ ⎥ ⎢ ⎥ ⎢ ⎥
⎢⎣ 0 ⎥⎦ ⎢⎣0⎥⎦ ⎢⎣0⎥⎦

=1

EA/L EA/L =1 =1

⎡− EA / L ⎤ ⎡0 ⎤ ⎡0 ⎤
⎢ 0 ⎥ ⎢0 ⎥ ⎢0 ⎥
⎢ ⎥ ⎢ ⎥ ⎢ ⎥
⎢ 0 ⎥ ⎢0 ⎥ ⎢0 ⎥
⎢ ⎥ ⎢ ⎥ ⎢ ⎥
⎢ EA / L ⎥ ⎢0 ⎥ ⎢0 ⎥
⎢ 0 ⎥ ⎢0 ⎥ ⎢0 ⎥
⎢ ⎥ ⎢ ⎥ ⎢ ⎥
⎢⎣ 0 ⎥⎦ ⎢⎣0⎥⎦ ⎢⎣0⎥⎦

EA / L 0 0 − EA / L 0 0
0 0 0 0 0 0
0 0 0 0 0 0
[ kl ] =
− EA / L 0 0 EA / L 0 0
0 0 0 0 0 0
0 0 0 0 0 0
SOLVED EXAMPLES
Example (1)

By using stiffness 3t/m


D2
matrix method, D3
D1
draw internal forces 2
2 3
diagrams for the
shown frame. 6t
3.00

EA = 4 * 103 t 1

EI = 5 * 104 t.m2 3.00

2.00 6.00

3t/m
Solution
Q D.O.F. = 3 3.6 5.4
∴ {F }3 x1 = [K ]3 x 3 {D}3 x1
3.0

3.0 2.7 6.3


⎡ −3 ⎤ 3.0

∴ {F }3x1 = ⎢⎢− 5.7 ⎥⎥


⎢⎣− 6.6⎥⎦
12
6.0

3.0 3.0

3.0

Element L θ c s EA/L 12EI/L3 6EI/L2 4EI/L 2EI/L


Ends
1 F-F 6.0 90 0 1 666.7 2777.78 8333.3 33333.3 16666.7
2 F-F 6.0 0 1 0 666.7 2777.78 8333.3 33333.3 16666.7
For element 1:
⎡ 666.7 0 0 − 666.7 0 0 ⎤
⎢ 0 2777.78 8333.3 0 − 2777.78 8333.3 ⎥⎥

⎢ 0 8333.3 33333.3 0 − 8333.3 16666.7 ⎥
[k1 ]l =⎢ ⎥
⎢− 666.7 0 0 666.7 0 0 ⎥
⎢ 0 − 2777.78 − 8333.3 0 2777.78 − 8333.3⎥
⎢ ⎥
⎢⎣ 0 8333.3 16666.7 0 − 8333.3 33333.3 ⎥⎦

⎡0 − 1 0 0 0 0⎤ ⎡0 1 0 0 0 0⎤
⎢1 0 0 0 0 0⎥⎥ ⎢− 1 0 0 0 0 0⎥⎥
⎢ ⎢
⎢0 0 1 0 0 0⎥ ⎢0 0 1 0 0 0⎥
[k1 ]g =⎢ ⎥ [k1 ]
l
⎢ ⎥
⎢0 0 0 0 −1 0⎥ ⎢0 0 0 0 1 0⎥
⎢0 0 0 1 0 0⎥ ⎢0 0 0 −1 0 0⎥
⎢ ⎥ ⎢ ⎥
⎢⎣0 0 0 0 0 1⎥⎦ ⎢⎣ 0 0 0 0 0 1⎥⎦

⎡ 2777.78 0 − 8333.3 − 2777.78 0 8333.3 ⎤


⎢ 0 666.7 0 0 − 666.7 0 ⎥⎥

⎢ − 8333.3 0 33333.3 8333.3 0 16666.7 ⎥
∴ [k1 ] =⎢
g

⎢− 2777.78 0 8333.3 2777.78 0 8333.3 ⎥
⎢ 0 − 666.7 0 0 666.7 0 ⎥
⎢ ⎥
⎢⎣ 8333.3 0 16666.7 8333.3 0 33333.3⎥⎦

⎡ 666.7 0 0 − 666.7 0 0 ⎤
⎢ 0 2777.78 8333.3 0 − 2777.78 8333.3 ⎥⎥

⎢ 0 8333.3 33333.3 0 − 8333.3 16666.7 ⎥
∴ [k 2 ] = [k 2 ]
g l
⎢ ⎥
⎢− 666.7 0 0 666.7 0 0 ⎥
⎢ 0 − 2777.78 − 8333.3 0 2777.78 − 8333.3⎥
⎢ ⎥
⎢⎣ 0 8333.3 16666.7 0 − 8333.3 33333.3 ⎥⎦
Overall equilibrium equation:

⎧ − 3 ⎫ ⎡3444.48 0 8333.3 ⎤ ⎧ D1 ⎫
⎪ ⎪ ⎢ ⎪ ⎪
⎨− 5.7 ⎬ = ⎢ 0 3444.48 8333.3 ⎥⎥ ⎨ D2 ⎬
⎪− 6.6⎪ ⎢ 8333.3 8333.3 66666.6⎥ ⎪ D ⎪
⎩ ⎭ ⎣ ⎦ ⎩ 3⎭

By solving 3 equations; then

⎧ D1 ⎫ ⎧− 2.19 × 10 ⎫
−3

⎪ ⎪ ⎪ −3 ⎪
⎨ D2 ⎬ = ⎨− 2.98 × 10 ⎬
⎪ D ⎪ ⎪ 5.48 × 10 − 4 ⎪
⎩ 3⎭ ⎩ ⎭

For member 1:

⎧ 0 ⎫ ⎧ 3 ⎫ ⎧ 4.516 ⎫
⎪ 0 ⎪ ⎪ 3 ⎪ ⎪ 4.986 ⎪
⎪ ⎪ ⎪ ⎪ ⎪ ⎪

⎪ 0 ⎪
⎪ ⎪
⎪ ⎪ ⎪
3 ⎪ ⎪ − 6. 11 ⎪⎪
{ F1 } g = [ k1 ] g ⎨ −3 ⎬
+⎨ ⎬=⎨ ⎬
⎪− 2.19 × 10 ⎪ ⎪− 3⎪ ⎪− 4.516⎪
⎪− 2.98 × 10 −3 ⎪ ⎪ 3 ⎪ ⎪ 1.014 ⎪
⎪ ⎪ ⎪ ⎪ ⎪ ⎪
⎪⎩ 5.48 × 10 − 4 ⎪⎭ ⎪⎩ 3 ⎪⎭ ⎪⎩ 3.016 ⎪⎭

For member 2:
⎧− 2.19 × 10 −3 ⎫ ⎧ 0 ⎫ ⎧ − 1.46 ⎫
⎪ −3 ⎪ ⎪ ⎪ ⎪ ⎪
⎪− 2.98 × 10 ⎪ ⎪ 2.7 ⎪ ⎪ − 1.01 ⎪
⎪ −4 ⎪
g ⎪ 5.48 × 10 ⎪ ⎪⎪ 3.6 ⎪⎪ ⎪⎪− 2.96⎪⎪
{ F2 } = [ k 2 ] ⎨ ⎬+⎨ ⎬=⎨
g

⎪ 0 ⎪ ⎪ 0 ⎪ ⎪ 1.46 ⎪
⎪ 0 ⎪ ⎪ 6.3 ⎪ ⎪ 10.01 ⎪
⎪ ⎪ ⎪ ⎪ ⎪ ⎪
⎪⎩ 0 ⎪⎭ ⎪⎩− 5.4⎪⎭ ⎪⎩ 21.09 ⎪⎭
Example 2: D3
D2 4t
D1
2
By using stiffness 6.0 t.m 2 4
2.00 1
matrix method, draw
4t
internal forces 1
diagrams for the 2.00 3
shown frame. 4t

EA = 4 * 103 t 2.00
3
EI = 5 * 104 t.m2
3.00 3.00 2.00 2.00

4t
Solution 1.5 2.0 2.0

Q D.O.F. = 3 6.0 t.m


1.5 2.0 2.0
1.5
∴ {F }3 x1 = [K ]3 x 3 {D}3 x1 5.33 4.0

1.5 4t
⎡ 4 ⎤
∴ {F }3 x1 = ⎢⎢− 0.5⎥⎥
⎢⎣ 1.83 ⎥⎦ 4t

5.33 4.0

Element L θ c s EA/L 12EI/L3 6EI/L2 4EI/L 2EI/L


Ends
1 F-F 6.32 18.43 0.99 0.316 632.91 2376.8 7510.81 31645.56 15822.78
2 F-F 4.0 0 1 0 1000 9375 18750 50000 25000
3 F-F 6.0 270 0 -1 666.67 2777.78 8333.3 33333.3 16666.7

For element 1:
⎡− − − − − − ⎤
⎢− − − − − − ⎥
⎢ ⎥
⎢− − − − − − ⎥
[k1 ]l =⎢ ⎥
⎢− − − 632.91 0 0 ⎥
⎢− − − 0 2376.8 − 7510.81⎥
⎢ ⎥
⎢⎣− − − 0 − 7510.81 31645.56 ⎥⎦
⎡ 0.99 − 0.316 0 0 0 0⎤ ⎡ 0.99 0.316 0 0 0 0⎤
⎢0.316 0.99 0 0 0 0⎥⎥ ⎢− 0.316 0.99 0 0 0 0⎥⎥
⎢ ⎢
⎢ 0 0 1 0 0 0⎥ ⎢ 0 0 1 0 0 0⎥
[k1 ]g =⎢ ⎥ [k1 ]
l
⎢ ⎥
⎢ 0 0 0 0.99 − 0.316 0⎥ ⎢ 0 0 0 0.99 0.316 0⎥
⎢ 0 0 0 0.316 0.99 0⎥ ⎢ 0 0 0 − 0.316 0.99 0⎥
⎢ ⎥ ⎢ ⎥
⎢⎣ 0 0 0 0 0 1⎥⎦ ⎢⎣ 0 0 0 0 0 1⎥⎦

⎡− − − − − − ⎤
⎢− − − − − − ⎥
⎢ ⎥
⎢− − − − − − ⎥
∴ [k1 ] =⎢
g

⎢− − − 857.65 − 545.55 2373.41 ⎥
⎢− − − − 545.55 2392.7 − 7435.7 ⎥
⎢ ⎥
⎢⎣− − − 2373.41 − 7435.7 31645.56⎥⎦

⎡1000 0 0 − − −⎤ ⎡2777.78 0 8333.3 − − −⎤


⎢ 0 9375 18750 − − −⎥⎥ ⎢ − − −⎥⎥
⎢ ⎢ 0 666.67 0
⎢ 0 18750 50000 − − −⎥ ⎢ 8333.3 0 33333.33 − − −⎥
∴ [k 2 ] = [k 2 ] ⎥ ∴ [k 3 ] = ⎢
g l g
⎢ ⎥
⎢ − − − − − −⎥ ⎢ − − − − − −⎥
⎢ − − − − − −⎥ ⎢ − − − − − −⎥
⎢ ⎥ ⎢ ⎥
⎢⎣ − − − − − −⎥⎦ ⎢⎣ − − − − − −⎥⎦

Overall equilibrium equation:

⎧ 4 ⎫ ⎡ 4635.35 − 545.55 10706.74 ⎤ ⎧ D1 ⎫


⎪ ⎪ ⎢ ⎥⎪ ⎪
⎨− 0.5⎬ = ⎢ − 545.55 12434.37 11314.3 ⎥ ⎨ D2 ⎬
⎪ 1.83 ⎪ ⎢10706.74 11314.3 114978.89⎥ ⎪ D ⎪
⎩ ⎭ ⎣ ⎦ ⎩ 3⎭

By solving 3 equations; then

⎧ D1 ⎫ ⎧ 1.09 × 10 ⎫
−3

⎪ ⎪ ⎪ −5 ⎪
⎨ D2 ⎬ = ⎨ 9.44 × 10 ⎬
⎪ D ⎪ ⎪− 9.52 × 10 −5 ⎪
⎩ 3⎭ ⎩ ⎭
For member 1:
⎧ 0 ⎫ ⎧ 0 ⎫ ⎧− 0.675⎫
⎪ 0 ⎪ ⎪ 1.5 ⎪ ⎪ 1.161 ⎪
⎪ ⎪ ⎪ ⎪ ⎪ ⎪

g ⎪ 0 ⎪⎪ ⎪⎪ 1.5 ⎪⎪ ⎪⎪ 1.878 ⎪⎪
{ F1 } = [ k1 ] ⎨
g
−3 ⎬
+⎨ ⎬=⎨ ⎬
⎪ 1.09 × 10 ⎪ ⎪ 0 ⎪ ⎪ 0.675 ⎪
⎪ 9.44 × 10 −5 ⎪ ⎪− 1.5⎪ ⎪ − 1.61 ⎪
⎪ ⎪ ⎪ ⎪ ⎪ ⎪
⎪⎩− 9.52 × 10 −5 ⎪⎭ ⎪⎩ 1.5 ⎪⎭ ⎪⎩ 0.373 ⎪⎭

For member 2:
⎧ 1.09 × 10 −3 ⎫ ⎧ 0 ⎫ ⎧ 1.09 ⎫
⎪ −5 ⎪ ⎪ ⎪ ⎪ ⎪
⎪ 9.44 × 10 ⎪ ⎪ 2.0 ⎪ ⎪ 1.10 ⎪
⎪⎪− 9.52 × 10 ⎪⎪ ⎪⎪ 2.0 ⎪⎪ ⎪⎪− 0.99⎪⎪
−5
{ F2 } g = [ k 2 ] g ⎨ ⎬+⎨ ⎬=⎨ ⎬
⎪ 0 ⎪ ⎪ 0 ⎪ ⎪ − 1.09 ⎪
⎪ 0 ⎪ ⎪ 2.0 ⎪ ⎪ 2.90 ⎪
⎪ ⎪ ⎪ ⎪ ⎪ ⎪
⎪⎩ 0 ⎪⎭ ⎪⎩− 2.0⎪⎭ ⎪⎩− 6.15⎪⎭

For member 3:
⎧ 1.09 × 10 −3 ⎫ ⎧ − 4 ⎫ ⎧ − 1.77 ⎫
⎪ −5 ⎪ ⎪ ⎪ ⎪ ⎪
⎪ 9.44 × 10 ⎪ ⎪ 0 ⎪ ⎪ 0.062 ⎪
⎪ −5 ⎪
g ⎪− 9.52 × 10 ⎪
⎪⎪− 5.33⎪⎪ ⎪⎪ 0.57 ⎪⎪
{ F3 } = [ k 3 ] ⎨ ⎬+⎨ ⎬=⎨
g

⎪ 0 ⎪ ⎪ − 4 ⎪ ⎪ − 6.23 ⎪
⎪ 0 ⎪ ⎪ 0 ⎪ ⎪− 0.062⎪
⎪ ⎪ ⎪ ⎪ ⎪ ⎪
⎪⎩ 0 ⎪⎭ ⎪⎩ 5.33 ⎪⎭ ⎪⎩ 12.82 ⎪⎭

Example 3:
2t/m
By using stiffness matrix D6
D5
D4
method, draw internal
4
forces diagrams for the 3 4
8.0 t.m
shown frame. 3.00 2
4t
D2
D3
3 D1
EA = 4 * 10 t 8t
2
EI = 5 * 104 t.m2
1
3.00 3

1 5

2.00 2.00 2.00 4.00


Solution 2t/m
4.8 4.8
Q D.O.F. = 3 2.0
2.0

∴ {F }3 x1 = [K ]3 x 3 {D}3 x1 8.0 t.m 2.0 4.0 4.0


3.0
2.0

2.0
⎡ 4 ⎤
= ⎢⎢− 0.5⎥⎥
8.0
∴ {F }3 x1
4.0
3.0

⎢⎣ 1.83 ⎥⎦
0.0

2.0 2.0

2.0

Element L θ c s EA/L 12EI/L3 6EI/L2 4EI/L 2EI/L


Ends
1 F-F 3.0 90 0 1 1333.34 22222.2 33333.34 66666.67 33333.34
2 F-F 5.0 36.87 0.8 0.6 800 4800 12000 40000 20000
3 F-F 6.0 270 0 -1 666.67 2777.78 8333.34 33333.34 16666.67
4 F-F 6.0 0 1 0 666.67 2777.78 8333.34 33333.34 16666.67

For element 1:
⎡ 1333.34 0 0 − 1333.34 0 0 ⎤
⎢ 0 22222.2 33333.34 0 − 22222.2 33333.34 ⎥⎥

⎢ 0 33333.34 66666.67 0 − 33333.34 33333.34 ⎥
[k1 ]l =⎢ ⎥
⎢− 1333.34 0 0 1333.34 0 0 ⎥
⎢ 0 − 22222.2 − 33333.34 0 22222.2 − 33333.34⎥
⎢ ⎥
⎢⎣ 0 33333.34 33333.34 0 − 33333.34 66666.67 ⎥⎦

⎡0 − 1 0 0 0 0⎤ ⎡0 1 0 0 0 0⎤
⎢1 0 0 0 0 0⎥⎥ ⎢− 1 0 0 0 0 0⎥⎥
⎢ ⎢
⎢0 0 1 0 0 0⎥ ⎢0 0 1 0 0 0⎥
[k1 ]g =⎢ ⎥ [k1 ]
l
⎢ ⎥
⎢0 0 0 0 −1 0⎥ ⎢0 0 0 0 1 0⎥
⎢0 0 0 1 0 0⎥ ⎢0 0 0 −1 0 0⎥
⎢ ⎥ ⎢ ⎥
⎢⎣0 0 0 0 0 1⎥⎦ ⎢⎣ 0 0 0 0 0 1⎥⎦
⎡ 22222.2 0 − 33333.34 − 22222.2 0 − 33333.34⎤
⎢ 0 1333.34 0 0 − 1333.34 0 ⎥
⎢ ⎥
⎢− 33333.34 0 66666.67 33333.34 0 33333.34 ⎥
∴ [k1 ] =⎢
g

⎢ − 22222.2 0 33333.3 22222.2 0 33333.34 ⎥
⎢ 0 − 1333.34 0 0 1333.34 0 ⎥
⎢ ⎥
⎢⎣− 33333.34 0 33333.34 33333.34 0 66666.67 ⎥⎦

For element 2:
⎡ 2777.78 0 8333.34 − 2777.78 0 8333.34 ⎤
⎢ 0 666.67 0 0 − 666.67 0 ⎥
⎢ ⎥
⎢ 8333.34 0 33333.34 − 8333.34 0 16666.67 ⎥
∴ [k 2 ] =⎢
g

⎢− 2777.78 0 − 8333.34 2777.78 0 − 8333.34⎥
⎢ 0 − 666.67 0 0 666.67 0 ⎥
⎢ ⎥
⎢⎣ 8333.34 0 16666.67 − 8333.34 0 33333.34 ⎥⎦

For element 3:
⎡ 2777.78 0 8333.34 − 2777.78 0 8333.34 ⎤
⎢ 0 666.67 0 0 − 666.67 0 ⎥
⎢ ⎥
⎢ 8333.34 0 33333.34 − 8333.34 0 16666.67 ⎥
∴ [k 3 ] =⎢
g

⎢− 2777.78 0 − 8333.34 2777.78 0 − 8333.34⎥
⎢ 0 − 666.67 0 0 666.67 0 ⎥
⎢ ⎥
⎢⎣ 8333.34 0 16666.67 − 8333.34 0 33333.34 ⎥⎦

For element 4:
⎡ 666.67 0 0 − 666.67 0 0 ⎤
⎢ 0 2777.78 8333.34 0 − 2777.78 8333.34 ⎥⎥

⎢ 0 8333.34 33333.34 0 − 8333.34 16666.67 ⎥
∴ [k 4 ] = [k 4 ] =⎢
g l

⎢− 666.67 0 0 666.67 0 0 ⎥
⎢ 0 − 2777.78 − 8333.34 0 2777.78 − 8333.34⎥
⎢ ⎥
⎢⎣ 0 8333.34 16666.67 0 − 8333.34 33333.34 ⎥⎦
Overall equilibrium equation:

⎧ 8 ⎫ ⎡ 24462.23 − 1920 26133.34 − 2240 1920 − 7200 ⎤ ⎧ D1 ⎫


⎪ − 3 ⎪ ⎢ − 1920 4693.34
⎪ ⎪ ⎢ 9600 1920 − 3360 9600 ⎥⎥ ⎪⎪ D2 ⎪⎪
⎪⎪ − 2 ⎪⎪ ⎢26133.34 9600 106666.67 7200 − 9600 20000 ⎥ ⎪⎪ D3 ⎪⎪
⎨ ⎬=⎢ ⎥⎨ ⎬
⎪ 2 ⎪ ⎢ − 2240 1920 7200 5684.44 − 1920 15533.34 ⎥ ⎪ D4 ⎪
⎪ − 3 ⎪ ⎢ 1920 − 3360 − 9600 − 1920 6804.44 − 1266.66 ⎥ ⎪ D5 ⎪
⎪ ⎪ ⎢ ⎥⎪ ⎪
⎪⎩− 4.8⎪⎭ ⎣⎢ 7200 − 9600 20000 15533.34 − 1266.66 106666.67 ⎦⎥ ⎪⎩ D6 ⎪⎭

By solving 6 equations; then

⎧ D1 ⎫ ⎧ 7.29 × 10 −4 ⎫
⎪D ⎪ ⎪ −3 ⎪
⎪ 2 ⎪ ⎪ − 1.02 × 10 ⎪
⎪⎪ D3 ⎪⎪ ⎪⎪ − 2.8 × 10 − 4 ⎪⎪
⎨ ⎬=⎨ −4 ⎬
⎪ D4 ⎪ ⎪ 9.9 × 10 ⎪
⎪ D5 ⎪ ⎪ − 1.275 × 10 −3 ⎪
⎪ ⎪ ⎪ ⎪
⎩⎪ D6 ⎭⎪ ⎪⎩− 2.929 × 10 ⎪⎭
−5

For member 1:
⎧ 0 ⎫ ⎧0⎫ ⎧− 6.84⎫
⎪ 0 ⎪ ⎪0⎪ ⎪ 1.36 ⎪
⎪ ⎪ ⎪ ⎪ ⎪ ⎪

⎪ 0 ⎪ ⎪
⎪ ⎪ ⎪ ⎪
0 ⎪ ⎪ 14. 94 ⎪⎪
{ F1 } g = [ k1 ] g ⎨ −4 ⎬ + ⎨ ⎬ = ⎨ ⎬
⎪ 7.29 × 10 ⎪ ⎪0⎪ ⎪ 6.84 ⎪
⎪ − 1.02 × 10 −3 ⎪ ⎪0⎪ ⎪ − 1.36 ⎪
⎪ ⎪ ⎪ ⎪ ⎪ ⎪
⎪⎩− 2.81 × 10 − 4 ⎪⎭ ⎪⎩0⎪⎭ ⎪⎩ 5.56 ⎪⎭

For member 2:
⎧ 7.29 × 10 −4 ⎫ ⎧ 0 ⎫ ⎧ 1.16 ⎫
⎪ −3 ⎪ ⎪ ⎪ ⎪ ⎪
⎪ − 1.02 × 10 ⎪ ⎪ 3 ⎪ ⎪ 1.379 ⎪
⎪⎪ − 2.81 × 10 − 4 ⎪⎪ ⎪⎪ 2 ⎪⎪ ⎪⎪ − 5.49 ⎪⎪
{ F2 } g = [ k 2 ] g ⎨ ⎬+⎨ ⎬=⎨ ⎬
⎪ ⎪ 0 ⎪ ⎪ − 1.16 ⎪
−4
⎪ 9.9 × 10
⎪ − 1.275 × 10 −3 ⎪ ⎪− 3⎪ ⎪ − 1.379 ⎪
⎪ ⎪ ⎪ ⎪ ⎪ ⎪
⎪⎩− 2.929 × 10 −5 ⎪⎭ ⎪⎩ 2 ⎪⎭ ⎪⎩− 0.4644⎪⎭
For member 3:
⎧ 9.9 × 10 −4 ⎫ ⎧− 2⎫ ⎧ 0.5 ⎫
⎪ −3 ⎪ ⎪ ⎪ ⎪ ⎪
⎪ − 1.275 × 10 ⎪ ⎪ 2 ⎪ ⎪ 1.15 ⎪
⎪⎪− 2.929 × 10 ⎪⎪ ⎪⎪− 2⎪⎪ ⎪⎪ 5.27 ⎪⎪
−5
{ F3 } g = [ k 3 ] g ⎨ ⎬+⎨ ⎬=⎨ ⎬
⎪ 0 ⎪ ⎪ 2 ⎪ ⎪ − 0.5 ⎪
⎪ 0 ⎪ ⎪− 2⎪ ⎪− 1.15⎪
⎪ ⎪ ⎪ ⎪ ⎪ ⎪
⎪⎩ 0 ⎪⎭ ⎪⎩− 2⎪⎭ ⎪⎩ 5.76 ⎪⎭

For member 4:
⎧ 9.9 × 10 −4 ⎫ ⎧ 0 ⎫ ⎧ 0.66 ⎫
⎪ −3 ⎪ ⎪ ⎪ ⎪ ⎪
⎪ − 1.275 × 10 ⎪ ⎪ 4 ⎪ ⎪ 0.215 ⎪
⎪ −5 ⎪
g ⎪− 2.929 × 10 ⎪
⎪⎪ 4.8 ⎪⎪ ⎪⎪ − 6.8 ⎪⎪
{ F4 } = [ k 4 ] ⎨ ⎬+⎨ ⎬=⎨
g

⎪ 0 ⎪ ⎪ 0 ⎪ ⎪− 0.66⎪
⎪ 0 ⎪ ⎪ 4 ⎪ ⎪ 7.78 ⎪
⎪ ⎪ ⎪ ⎪ ⎪ ⎪
⎪⎩ 0 ⎪⎭ ⎪⎩− 4.8⎪⎭ ⎪⎩ − 15.9 ⎪⎭

Example 4:
By using stiffness matrix D6
D5
D4
method, draw internal 1 3
1 2
forces diagrams for the 4

shown frame; if:

EA = 4 *103 t.
6.00
EI = 5 * 104 t.m2. 3

Δ2 = 2 cm.
Δt = 20° for member 2.
α = 1 * 10-5. 2t/m
2
h = 60 cm. 4.00 4.00
Solution 2.4
0.0 0.0
Q D.O.F. = 3 1.8

∴ {F }3 x1 = [K ]3 x 3 {D}3 x1

⎧1.8 ⎫
⎪ ⎪
∴ { f }3 x1 = ⎨ 0 ⎬ (due to loads)
⎪2.4⎪
⎩ ⎭ 2t/m 4.2
3.6

⎧ − 0.8 ⎫
⎪ ⎪
∴ { f }3 x1 = ⎨ 0 ⎬ (due to temp.) 16.67 16.67
0.8 t=0 0.8
⎪− 16.67 ⎪ 0.0 t = 20
⎩ ⎭

0.0

⎧ 0 ⎫
⎪ ⎪ 13.33
∴ { f }3 x1 = ⎨361.67 ⎬ (due to settle.)
375 375 375 375
⎪ 0 ⎪
⎩ ⎭

187.5 187.5 187.5 187.5

⎧ 1 ⎫
⎪ ⎪
∴ {F }3 x1 = ⎨ 361.67 ⎬
⎪− 14.27 ⎪
⎩ ⎭

13.33
Elemen L θ C s EA/L 12EI/L3 6EI/L2 4EI/L 2EI/L
t Ends
1 F-F 4.0 0 1 0 1000 9375 18750 50000 25000
2 F-F 4.0 180 -1 0 1000 9375 18750 50000 25000
3 F-F 6.0 90 0 1 666.67 2777.78 8333.33 33333.33 16666.7

For element 1:
⎡ 1000 0 0 − 1000 0 0 ⎤
⎢ 0 9375 18750 0 − 9375 18750 ⎥⎥

⎢ 0 18750 50000 0 − 18750 25000 ⎥
[k1 ]g = [k1 ]l =⎢ ⎥
⎢− 1000 0 0 1000 0 0 ⎥
⎢ 0 − 9375 − 18750 0 9375 − 18750⎥
⎢ ⎥
⎣⎢ 0 18750 25000 0 − 18750 50000 ⎦⎥

For element 2:
⎡ 1000 0 0 − 1000 0 0 ⎤
⎢ 0 9375 − 18750 0 − 9375 − 18750⎥⎥

⎢ 0 − 18750 50000 0 18750 25000 ⎥
[k 2 ]g = [k 2 ]l =⎢ ⎥
⎢− 1000 0 0 1000 0 0 ⎥
⎢ 0 − 9375 18750 0 9375 18750 ⎥
⎢ ⎥
⎢⎣ 0 − 18750 25000 0 18750 50000 ⎥⎦

For element 3:
⎡ 2777.78 0 − 8333.3 − 2777.78 0 − 8333.3⎤
⎢ 0 666.7 0 0 − 666.7 0 ⎥⎥

⎢ − 8333.3 0 33333.3 8333.3 0 16666.7 ⎥
∴ [k 3 ] =⎢
g

⎢− 2777.78 0 8333.3 2777.78 0 8333.3 ⎥
⎢ 0 − 666.7 0 0 666.7 0 ⎥
⎢ ⎥
⎢⎣ − 8333.3 0 16666.7 8333.3 0 33333.3 ⎥⎦

Overall equilibrium equation:

⎧ 1 ⎫ ⎡4777.78 0 8333.3 ⎤ ⎧ D1 ⎫
⎪ ⎪ ⎢ ⎥ ⎪D ⎪
⎨ 361.67 ⎬ = ⎢ 0 194166.67 0 ⎥ ⎨ 2⎬
⎪− 14.27 ⎪ ⎢ 8333.3
⎩ ⎭ ⎣ 0 133333.33⎥⎦ ⎪⎩ D3 ⎪⎭

By solving 3 equations; then


⎧ D1 ⎫ ⎧ 4.44 × 10 ⎫
−4

⎪ ⎪ ⎪ −3 ⎪
⎨ D2 ⎬ = ⎨ 1.86 × 10 ⎬
⎪ D ⎪ ⎪− 1.348 × 10 − 4 ⎪
⎩ 3⎭ ⎩ ⎭
For member 1:
⎧ 0 ⎫ ⎧ 0 ⎫ ⎧ − 0.44 ⎫
⎪ 0 ⎪ ⎪ 187.5 ⎪ ⎪ 167.51 ⎪
⎪ ⎪ ⎪ ⎪ ⎪ ⎪

⎪ 0 ⎪
⎪ ⎪⎪ 375 ⎪
⎪ ⎪⎪ 336 . 7 ⎪⎪
{ F1 } g = [ k1 ] g ⎨ −4 ⎬
+⎨ ⎬=⎨ ⎬
⎪ 4.44 × 10 ⎪ ⎪ 0 ⎪ ⎪ 0.44 ⎪
⎪ 1.86 × 10 −3 ⎪ ⎪− 187.5⎪ ⎪− 167.51⎪
⎪ ⎪ ⎪ ⎪ ⎪ ⎪
⎪⎩− 1.348 × 10 − 4 ⎪⎭ ⎪⎩ 375 ⎪⎭ ⎪⎩ 333.33 ⎪⎭

For member 2:
⎧ 0 ⎫ ⎧ − 0.8 ⎫ ⎧ 0 ⎫ ⎧ − 1.24 ⎫
⎪ 0 ⎪ ⎪ 0 ⎪ ⎪ 187.5 ⎪ ⎪ 172.56 ⎪
⎪ ⎪ ⎪ ⎪ ⎪ ⎪ ⎪ ⎪

⎪ 0 ⎪
⎪ ⎪⎪ − 16 . 67 ⎪ ⎪
⎪ ⎪ − 375 ⎪ ⎪
⎪ ⎪ − 360 . 11⎪⎪
{ F2 } g = [ k 2 ] g ⎨ −4 ⎬
+⎨ ⎬+⎨ ⎬=⎨ ⎬
⎪ 4.44 × 10 ⎪ ⎪ 0.8 ⎪ ⎪ 0 ⎪ ⎪ 1.24 ⎪
⎪ 1.86 × 10 −3 ⎪ ⎪ 0 ⎪ ⎪− 187.5⎪ ⎪ − 172.56 ⎪
⎪ ⎪ ⎪ ⎪ ⎪ ⎪ ⎪ ⎪
⎪⎩− 1.348 × 10 − 4 ⎪⎭ ⎪⎩ 16.67 ⎪⎭ ⎪⎩ − 375 ⎪⎭ ⎪⎩− 330.14⎪⎭

For member 3:
⎧ 0 ⎫ ⎧ − 3.8⎫ ⎧ 0 ⎫ ⎧ − 3.9 ⎫
⎪ 0 ⎪ ⎪ 0 ⎪ ⎪− 13.33⎪ ⎪− 14.53⎪
⎪ ⎪ ⎪ ⎪ ⎪ ⎪ ⎪ ⎪

⎪ 0 ⎪
⎪ ⎪ ⎪ 3 . 6 ⎪
⎪ ⎪ ⎪ 0 ⎪
⎪ ⎪ ⎪ 5 . 06 ⎪⎪
{ F3 } g = [ k 3 ] g ⎨ −4 ⎬ + ⎨ ⎬ + ⎨ ⎬ = ⎨ ⎬
⎪ 4.44 × 10 ⎪ ⎪ − 1.8 ⎪ ⎪ 0 ⎪ ⎪ − 1.69 ⎪
⎪ 1.86 × 10 −3 ⎪ ⎪ 0 ⎪ ⎪ 13.33 ⎪ ⎪ 14.57 ⎪
⎪ ⎪ ⎪ ⎪ ⎪ ⎪ ⎪ ⎪
⎪⎩− 1.348 × 10 − 4 ⎪⎭ ⎪⎩− 2.4⎪⎭ ⎪⎩ 0 ⎪⎭ ⎪⎩ − 3.19 ⎪⎭

Example 5:
6 t.m 3t
1 2t 3
By using stiffness matrix 1 2 2
method, draw internal
forces diagrams for the
shown frame. 3
5.00

4.00 4.00
Solution
⎧2⎫
⎪ ⎪
Q D.O.F. = 3 ∴ {F }3 x1 = [K ]3 x 3 {D}3 x1 ∴ {F }3 x1 = ⎨− 3⎬
⎪6⎪
⎩ ⎭
Elemen L Θ c s EA/L 12EI/L3 6EI/L2 4EI/L 2EI/L
t Ends
1 F-F 4.0 0 1 0 1000 9375 18750 50000 25000
2 F-F 4.0 0 1 0 1000 9375 18750 50000 25000
3 F-F 5.0 270 0 -1 800 4800 12000 40000 20000

For element 1:
⎡− − − − − − ⎤
⎢− − − − − − ⎥⎥

⎢− − − − − − ⎥
[k1 ]g = [k1 ]l =⎢ ⎥
⎢− − − 1000 0 0 ⎥
⎢− − − 0 9375 − 18750⎥
⎢ ⎥
⎢⎣− − − 0 − 18750 50000 ⎥⎦

For element 2:
⎡1000 0 0 − − −⎤
⎢ 0 9375 18750 − − −⎥⎥

⎢ 0 18750 50000 − − −⎥
[k 2 ]g = [k 2 ]l =⎢ ⎥
⎢ − − − − − −⎥
⎢ − − − − − −⎥
⎢ ⎥
⎢⎣ − − − − − −⎥⎦
For element 3:
⎡800 0 0 − − −⎤
⎢ 0 9375 18750 − − −⎥⎥

⎢ 0 18750 50000 − − −⎥
∴ [k 3 ] =⎢
l

⎢ − − − − − −⎥
⎢ − − − − − −⎥
⎢ ⎥
⎢⎣ − − − − − −⎥⎦

⎡0 1 0 0 0 0⎤ ⎡0 − 1 0 0 0 0⎤
⎢− 1 0 0 0 0 0⎥⎥ ⎢1 0 0 0 0 0⎥⎥
⎢ ⎢
⎢0 0 1 0 0 0⎥ ⎢0 0 1 0 0 0⎥
∴ [k 3 ] =⎢ ⎥ [k 3 ]
g l
⎢ ⎥
⎢0 0 0 0 1 0⎥ ⎢0 0 0 0 − 1 0⎥
⎢0 0 0 − 1 0 0⎥ ⎢0 0 0 1 0 0⎥
⎢ ⎥ ⎢ ⎥
⎣⎢ 0 0 0 0 0 1⎦⎥ ⎣⎢0 0 0 0 0 1⎦⎥

⎡ 4800 0 12000 − − −⎤
⎢ 0 800 0 − − −⎥⎥

⎢12000 0 40000 − − −⎥
∴ [k 3 ] =⎢
g

⎢ − − − − − −⎥
⎢ − − − − − −⎥
⎢ ⎥
⎢⎣ − − − − − −⎥⎦

Overall equilibrium equation:

⎧ 2 ⎫ ⎡ 6800 0 12000 ⎤ ⎧ D1 ⎫
⎪ ⎪ ⎢ ⎪ ⎪
⎨− 3⎬ = ⎢ 0 19550 0 ⎥⎥ ⎨ D2 ⎬
⎪ 6 ⎪ ⎢12000
⎩ ⎭ ⎣ 0 140000⎥⎦ ⎪⎩ D3 ⎪⎭

By solving 3 equations; then

⎧ D1 ⎫ ⎧ 2.57 × 10 ⎫
−4

⎪ ⎪ ⎪ −4 ⎪
⎨ D2 ⎬ = ⎨− 1.53 × 10 ⎬
⎪ D ⎪ ⎪ 2.07 × 10 −5 ⎪
⎩ 3⎭ ⎩ ⎭
For member 1:

⎡− − − − 1000 0 0 ⎤⎧ 0 ⎫ ⎧0⎫ ⎧− 0.257 ⎫


⎢− − − 0 ⎥
− 9375 18750 ⎥ ⎪ ⎪ 0 ⎪ ⎪0⎪ ⎪ 1.822 ⎪
⎢ ⎪ ⎪ ⎪ ⎪ ⎪
⎢− − − 0 − 18750 25000 ⎥ ⎪⎪ 0 ⎪⎪ ⎪⎪0⎪⎪ ⎪⎪ 3.38 ⎪⎪
{ F1 } = ⎢
g
⎥⎨ ⎬+⎨ ⎬=⎨ ⎬
⎢− − − 1000 0 0 ⎥ ⎪ 2.57 × 10 − 4 ⎪ ⎪0⎪ ⎪ 0.257 ⎪
⎢− − − 0 9375 − 18750⎥ ⎪− 1.53 × 10 − 4 ⎪ ⎪0⎪ ⎪ − 1.822 ⎪
⎢ ⎥⎪ ⎪ ⎪ ⎪ ⎪ ⎪
⎢⎣− − − 0 − 18750 50000 ⎥⎦ ⎪⎩ 2.07 × 10 −5 ⎪⎭ ⎪⎩0⎪⎭ ⎪⎩ 3.9 ⎪⎭

For member 2:

⎡ 1000 0 0 − − − ⎤ ⎧ 2.57 × 10 −4 ⎫ ⎧0⎫ ⎧ 0.257 ⎫


⎪ ⎪
⎢ 0
⎢ 9375 18750 − − − ⎥⎥ ⎪− 1.53 × 10 − 4 ⎪ ⎪⎪0⎪⎪ ⎪⎪ − 1.046 ⎪⎪
⎢ 0 18750 50000 − − − ⎥ ⎪⎪ 2.07 × 10 −5 ⎪⎪ ⎪⎪0⎪⎪ ⎪⎪ − 1.833 ⎪⎪
{ F2 } g = ⎢ ⎥⎨ ⎬+⎨ ⎬=⎨ ⎬
⎢− 1000 0 0 − − −⎥ ⎪ 0 ⎪ ⎪0⎪ ⎪− 0.257 ⎪
⎢ 0 − 9375 − 18750 − − −⎥ ⎪ 0 ⎪ ⎪0⎪ ⎪ 1.04 ⎪
⎢ ⎥⎪ ⎪ ⎪ ⎪ ⎪ ⎪
⎣⎢ 0 18750 25000 − − − ⎦⎥ ⎪⎩ 0 ⎪⎭ ⎪⎩0⎪⎭ ⎪⎩ − 2.35 ⎪⎭

For member 3:

⎡ 4800 0 12000 − − − ⎤ ⎧ 2.57 × 10 −4 ⎫ ⎧0⎫ ⎧ 1.482 ⎫


⎪ ⎪
⎢ 0
⎢ 800 0 − − − ⎥⎥ ⎪− 1.53 × 10 − 4 ⎪ ⎪⎪0⎪⎪ ⎪⎪− 0.122⎪⎪
⎢ 12000 0 40000 − − − ⎥ ⎪⎪ 2.07 × 10 −5 ⎪⎪ ⎪⎪0⎪⎪ ⎪⎪ 3.91 ⎪⎪
{ F3 } g = ⎢ ⎥⎨ ⎬+⎨ ⎬=⎨ ⎬
⎢− 4800 0 − 12000 − − −⎥ ⎪ 0 ⎪ ⎪0⎪ ⎪ − 1.48 ⎪
⎢ 0 − 800 0 − − −⎥ ⎪ 0 ⎪ ⎪0⎪ ⎪ 0.122 ⎪
⎢ ⎥⎪ ⎪ ⎪ ⎪ ⎪ ⎪
⎢⎣ 12000 0 20000 − − − ⎥⎦ ⎪⎩ 0 ⎪⎭ ⎪⎩0⎪⎭ ⎪⎩ 3.49 ⎪⎭
Example 6: 2t/m 2t/m
5t
1
By using stiffness 1 2
3 4
matrix method, draw
2.00
internal forces 4 4t
diagrams for the
shown frame. 2.00
3

3.00 3.00 3.00 3.00

2t/m 2t/m
Solution 3.75 5t 3.75
2.25 2.25
Q D.O.F. = 5
∴ {F }5×1 = [K ]5×5 {D}5×1
2.0
2.5 2.5 3.0 3.0
2.0
⎧− 3.75⎫
⎪ 2 ⎪ 4t
⎪⎪ ⎪⎪ 0.0
∴ {F }5×1 = ⎨ −2 ⎬
⎪ − 5.5 ⎪ 2.0
⎪ ⎪
⎪⎩ − 0.5 ⎪⎭ 2.0

Elemen L Θ c s EA/L 12EI/L3 6EI/L2 4EI/L 2EI/L


t Ends
1 F-F 6 0 1 0 666.67 2777.78 8333.33 33333.33 16666.67
2 F-F 6 0 1 0 666.67 2777.78 8333.33 33333.33 16666.67
3 F-F 4 90 0 1 1000 9375 18750 50000 25000
For element 1:
⎡ 666.67 0 0 − 666.67 0 0 ⎤
⎢ 0 2777.78 8333.33 0 − 2777.78 8333.33 ⎥⎥

⎢ 0 8333.33 33333.33 0 − 8333.33 16666.67 ⎥
[k1 ]g = [k1 ]l =⎢ ⎥
⎢− 666.67 0 0 666.67 0 0 ⎥
⎢ 0 − 2777.78 − 8333.33 0 2777.78 − 8333.33⎥
⎢ ⎥
⎢⎣ 0 8333.33 16666.67 0 − 8333.33 33333.33 ⎥⎦

For element 2:
⎡ 666.67 0 0 − 666.67 0 0 ⎤
⎢ 0 2777.78 8333.33 0 − 2777.78 8333.33 ⎥⎥

⎢ 0 8333.33 33333.33 0 − 8333.33 16666.67 ⎥
[k1 ]g = [k1 ]l =⎢ ⎥
⎢− 666.67 0 0 666.67 0 0 ⎥
⎢ 0 − 2777.78 − 8333.33 0 2777.78 − 8333.33⎥
⎢ ⎥
⎢⎣ 0 8333.33 16666.67 0 − 8333.33 33333.33 ⎥⎦

For element 3:
⎡ 9375 0 − 18750 − 9375 0 − 18750⎤
⎢ 0 1000 0 0 − 1000 0 ⎥⎥

⎢− 18750 0 50000 18750 0 25000 ⎥
∴ [k 3 ] =⎢
g

⎢ − 9375 0 18750 9375 0 18750 ⎥
⎢ 0 − 1000 0 0 1000 0 ⎥
⎢ ⎥
⎢⎣ 18750 0 25000 18750 − 50000 ⎥⎦

Overall equilibrium equation:

⎧− 3.75⎫ ⎡ 33333.33 0 0 − 8333.33 16666.67 ⎤ ⎧ D1 ⎫


⎪ 2 ⎪ ⎢
⎪⎪ ⎪⎪ ⎢ 0 50000 18750 0 25000 ⎥⎥ ⎪⎪ D2 ⎪⎪
⎪ ⎪
⎨ −2 ⎬= ⎢ 0 18750 10708.34 0 18750 ⎥ ⎨ D3 ⎬
⎪ − 5.5 ⎪ ⎢− 8333.33 ⎥⎪ ⎪
⎪ ⎪ ⎢
0 0 6555.56 0 ⎥ ⎪ D4 ⎪
⎪⎩ − 0.5 ⎪⎭ ⎢⎣ 16666.67 25000 18750 0 16666.67 ⎥⎦ ⎪⎩ D5 ⎪⎭

By solving 5 equations; then


⎧ D1 ⎫ ⎧ 0.0 ⎫
⎪D ⎪ ⎪ 0.0 ⎪
⎪⎪ 2 ⎪⎪ ⎪⎪ ⎪
−3 ⎪
⎨ 3⎬ ⎨
D = − 1 . 6559 × 10 ⎬
⎪ D ⎪ ⎪ − 1.96 × 10 −3 ⎪
⎪ 4⎪ ⎪ ⎪
⎪⎩ D5 ⎪⎭ ⎪⎩ 2.76 × 10 −3 ⎪⎭
For member 1:
⎧ 0 ⎫ ⎧ 1.104 ⎫
⎪ 0 ⎪ ⎪ 13.455 ⎪
⎪ ⎪ ⎪ ⎪
⎪⎪ 0 ⎪⎪ ⎪⎪ 0 ⎪⎪
{ F1 } g = [k1 ] + {F1 } = ⎨
g l
⎨ −3 ⎬ ⎬
⎪− 1.6559 × 10 ⎪ ⎪ − 1.104 ⎪
⎪ − 1.96 × 10 −3 ⎪ ⎪− 8.455⎪
⎪ ⎪ ⎪ ⎪
⎪⎩ 2.76 × 10 −3 ⎪⎭ ⎪⎩ 65.74 ⎪⎭

Similarly:
⎧ − 1.104 ⎫ ⎧ − 21.5 ⎫
⎪− 20.72⎪ ⎪1.9064⎪
⎪ ⎪ ⎪ ⎪

⎪ 78 . 42 ⎪⎪ ⎪
⎪ 0 ⎪⎪
{ F2 } g = ⎨ ⎬ { F3 } g = ⎨ ⎬
⎪ 1.104 ⎪ ⎪ 25.5 ⎪
⎪ 14.72 ⎪ ⎪1.9064⎪
⎪ ⎪ ⎪ ⎪
⎪⎩ 27.89 ⎪⎭ ⎪⎩ 94 ⎪⎭

Example 7:
Construct the equilibrium 6t
equation of the following D3
D2
D6
D5
2.00 2t 2t
frame. D1 2 t / m D4

2 1 3 1 4
3
EA = 3 x 10 t.
EI = 4 x 104 t.m2.
6.00 4
3

4t/m 1

3.00 3.00 3.00 3.00 2.00

3.0 12.0 3.0 2t


Solution 3.0 3.0 7.5
2t/m 7.5
6.0
Q D.O.F. = 6
∴ {F }6×1 = [K ]6×6 {D}6×1
3.0 3.0 4.8 7.0 7.0
3.6

⎧ 6.6 ⎫
⎪− 10⎪
⎪ ⎪
⎪⎪ 0.3 ⎪⎪
∴ {F }6×1 =⎨ ⎬
⎪ 0 ⎪
⎪−9 ⎪
⎪ ⎪ 4t/m 8.4
⎪⎩ 3.5 ⎪⎭ 7.2
Element L Θ c S EA/L 12EI/L3 6EI/L2 4EI/L 2EI/L
Ends
1 F-F 6 0 1 0 500 222.22 666.67 2666.67 1333.33
2 F-F 6 0 1 0 500 222.22 666.67 2666.67 1333.33
3 F-F 6 90 0 1 500 222.22 666.67 2666.67 1333.33

For element 1:
⎡− − − − − ⎤ −
⎢− − − − − − ⎥⎥

⎢− − − − − − ⎥
[k1 ]g = [k1 ]l =⎢ ⎥
⎢− − − 500 0 0 ⎥
⎢− − − 0 222.22 − 666.67⎥
⎢ ⎥
⎣⎢− − − 0 − 666.67 2666.67 ⎦⎥
For element 2:
⎡ 500 0 0 − 500 0 0 ⎤
⎢ 0 222.22 666.67 0 − 222.22 666.67 ⎥⎥

⎢ 0 666.67 2666.67 0 − 666.67 1333.33 ⎥
[k 2 ]g = [k 2 ]l =⎢ ⎥
⎢− 500 0 0 500 0 0 ⎥
⎢ 0 − 222.22 − 666.67 0 222.22 − 666.67⎥
⎢ ⎥
⎣⎢ 0 666.67 1333.33 0 − 666.67 2666.67 ⎦⎥

⎡ − − − − − − ⎤
⎢ − − − − − − ⎥⎥

⎢ − − − − − − ⎥
∴ [k 4 ] = ⎢−
g

⎢ − − − 215.84 137.42 235.40 ⎥
⎢ − − − 137.42 215.84 − 235.40⎥
⎢ ⎥
⎣⎢ − − − 235.40 − 235.40 1884.57 ⎦⎥

Overall equilibrium equation:

⎧ 6.6 ⎫ ⎡1222.22 0 666.67 − 500 0 0 ⎤ ⎧ D1 ⎫


⎪− 10⎪ ⎢ 0
⎪ ⎪ ⎢ 944.44 0 0 − 222.22 666.67 ⎥⎥ ⎪⎪ D2 ⎪⎪
⎪⎪ 0.3 ⎪⎪ ⎢ 666.67 0 8000.01 0 − 666.67 1333.33 ⎥ ⎪⎪ D3 ⎪⎪
⎨ ⎬=⎢ ⎥⎨ ⎬
⎪ 0 ⎪ ⎢ − 500 0 0 853.36 0 0 ⎥ ⎪ D4 ⎪
⎪−9 ⎪ ⎢ 0 − 222.22 − 666.67 0 300.66 − 999.63⎥ ⎪ D5 ⎪
⎪ ⎪ ⎢ ⎥⎪ ⎪
⎪⎩ 3.5 ⎪⎭ ⎢⎣ 0 666.67 1333.33 0 − 999.63 4551.24 ⎥⎦ ⎪⎩ D6 ⎪⎭

You might also like